Infectious diseases RACP MCQs Flashcards

1
Q

RACP 2022 Q 12

Yersinia Pestis caused the bubonic plague in Europe in the middle ages. What is the main vector for Yersinia pestis that transmits the disease to
humans?

A. Ticks
B. Fleas
C. Rats
D. Mosquitos

A

Answer B

Pathogen: Yersinia pestis
Reservoir: prairie dogs, squirrels, rodents
Route of transmission: flea bites

How well did you know this?
1
Not at all
2
3
4
5
Perfectly
2
Q

RACP 2022 Q 15
15.Rabies is now common in Bali. A patient returns from Bali and reports a monkey bite 3 weeks ago. He did not receive any treatment in Bali. What should you administer?

A. Rabies immunoglobulin and rabies vaccination now
B. Rabies immunoglobulin only
C. Rabies vaccine only
D. Do nothing as he was bitten three weeks ago

A

Answer A or C - depending on patient’s vaccination status

  • Post exposure prophylaxis - vaccination +/- rabies immunoglobulin ( Option B incorrect)
  • Incubation period is 45 days but can be longer, only late for PEP when patient starts displaying clinical signs of rabies (Option D incorrect)
  • If previously vaccinated against rabies - then PEP is only repeat rabies vaccination, if unvaccinated - then patient needs both vaccine and immunoglobulin
How well did you know this?
1
Not at all
2
3
4
5
Perfectly
3
Q

RACP 2022 Q16

16.75F with unilateral headache, photophobia, N+V with reddened left eye. What is the most appropriate initial investigation?

a. CT Brain
b. CT cerebral angiogram
c. Intraocular pressure measurement
d. Lumbar puncture

A

Answer C - quick and easy initial investigation to rule out acute closed-angle glaucoma

How well did you know this?
1
Not at all
2
3
4
5
Perfectly
4
Q

RACP 2022 Q 17
17.A patient reports one month history of deafness. Syphillis is suspected and his RPR positive, TPPA positive. What is the next appropriate step in management?

A. Benzathine benzylpenicillin 2.4 million units IM once-off
B. Benzathine benzylpenicillin 2.4 million units IM weekly for 3 weeks
C.Benzylpenicillin 1.8g IV 4hrly for 15 days
D. Procaine benzylpenicillin 1.5g IM daily for 10 days

A

Answer C

Otosyphilis is considered a manifestation of tertiary syphilis.

CDC guidelines: aqueous crystalline penicillin G 3-4mil units Q4h intravenously for 10-14 days

eTG: benzylpenicillin 1.8g q4h IV for 15 days

NB: aqueous crystalline penicillin G is also known as benzylpenicillin in Australia, 60mg of benpen is equivalent to 100,000 units of penicillin (i.e. 1.8g = 3mil units)

How well did you know this?
1
Not at all
2
3
4
5
Perfectly
5
Q

RACP 2022 Q 30

30ish male. Unprotected sex MSM 2 weeks ago. Presents with anal
discharge. Suspect infective proctitis. Sample sent for culture. Awaiting
result. What is the most appropriate empirical therapy?

A. Ceftriaxone + valacyclovir
B. Ceftriaxone + Doxycycline + valacyclovir
C. HIV PEP only
D. Nothing, wait for culture.

A

Answer B

Treat infective proctitis empirically as chlamydia, gonorrhoea, and HSV until culture results are known. Recommended treatment as per eTG is:
(1) ceftriaxone 500mg IM/IV once (N.gonorrhoea)
(2) doxycycline 100mg PO BD for 7 days (C.trachomatis)
(3) valaciclovir 500mg PO BD for 10 days (HSV)

How well did you know this?
1
Not at all
2
3
4
5
Perfectly
6
Q

RACP 2022 Q 50

  1. 25yo patient with restrictive VSD going for surgical wisdom tooth extraction.
    What infective endocarditis prophylaxis should they be given?

A. Amoxicillin 2g PO 30-60min before procedure
B. Clindamycin 600mg PO 30-60min before procedure
C. Cefalexin 1g PO 30-60min before procedure
D. No prophylaxis required

A

Answer A

Antibiotic prophylaxis regimens for invasive dental or invasive oral procedures target prevention of viridians group streptococcal IE. The preferred regimen is oral amoxicillin (adults: 2 grams; children: 50 mg/kg). Antibiotics should be administered 30 to 60 minutes prior to the procedure.

Clindamycin is no longer recommended as an alternative antibiotic regimen for patients undergoing dental procedures, given more frequent and severe adverse reactions associated with this drug compared with other antibiotic agents.

Cefalexin is indicated in patients with penicillin allergy.

How well did you know this?
1
Not at all
2
3
4
5
Perfectly
7
Q

RACP 2022 Q62

62.Patient presents with fever, headache, red eyes. No diarrhoea mentioned in stem. Recent travel history including swimming in freshwater lakes in Malaysia. What is the most likely organism?

A. Aeromonas
B. Vibrio (species named; not cholera)
C. Leptospirosis

A

Answer C

  • Leptospirosis is a zoonotic infection - caused by spirochetes that have a reservoir in rodents. Humans contract via exposure to infected rodent urine in soil or freshwater.
  • Causes icteric and anicteric (more common) illness. Latter is characterised by abrupt onset of fever, rigors, myalgias (especially in the calves and lower back), and headache. A distinguishing feature is conjunctival suffusion, which occurs in over half of patients.

Aeromonas consists of gram-negative rods widely distributed in freshwater, estuarine, and marine environments. Diarrheal disease is the most common manifestation of Aeromonas infection, but can also cause wound infections and bacteraemia.

Vibrio sp. cause gastrointestinal illness, wound infections and bacteraemia associated with salt water and seafood (contaminated oysters and raw shellfish).

How well did you know this?
1
Not at all
2
3
4
5
Perfectly
8
Q

RACP 2022a Q88

88.EMQ: Which of the following causative organisms of infective endocarditis is
culture negative?

a. Coxiella burnetti
b. Strep gallolyticus
c. Haemophilus influenza
d. Acetinobacter Baumanii
e. Staph aureus
f. Aspergillus

A

Answer A and F

Coxiella burnetti and Bartonella spp. are fastidious organisms which require very specific conditions to grow and thus may not be positive on blood cultures.

Fungal bacteremia are also often negative on routine blood cultures.

How well did you know this?
1
Not at all
2
3
4
5
Perfectly
9
Q

RACP 2022a Q87

87.EMQ: Which of the following causative organisms of infective endocarditis is
associated with bowel cancer?
a. Coxiella burnetti
b. Strep gallolyticus
c. Haemophilus influenza
d. Acetinobacter Baumanii
e. Staph aureus
f. Aspergillus

A

Answer B

Streptococcus gallolyticus ( formerly known as Strep bovis) - associated with bowel cancer.

Patients with endocarditis secondary to S. bovis group should be investigated for a concurrent bowel neoplasm.

How well did you know this?
1
Not at all
2
3
4
5
Perfectly
10
Q

RACP 2022 P2 Q11

  1. How does probenecid affect the clearance of cephazolin?

A. Increased renal tubular reabsorption
B. Increased glomerular filtration
C. decreased tubular secretion

A

Answer C

Probenecid decreases secretion of beta-lactam antibiotics in the proximal convoluted tubule.

How well did you know this?
1
Not at all
2
3
4
5
Perfectly
11
Q

RACP 2022 P2 Q23

  1. What is the appearance of Listeria monogenes on gram stain?

A. Gram positive cocci
B. Gram negative rod
C. Gram positive rod
D. Gram negative coccobacilli

A

Answer C Gram positive rod

How well did you know this?
1
Not at all
2
3
4
5
Perfectly
12
Q

RACP 2022 Q56

  1. What is the most common cause of cellulitis?

A. Group A streptococcus
B. Staph aureus
C. Staph epidermidis

A

Answer A

Most common cause of cellulitis and erisypelas is beta-haemolytic strep (group A strep e.g. S.pyogenes) followed by MSSA

How well did you know this?
1
Not at all
2
3
4
5
Perfectly
13
Q

RACP 2021 Q2

Q2. A 53 year old woman presents with sudden onset joint pain and swelling. She is febrile with a WCC of 13. Joint aspirate shows 103,000 WCC with >95% neutrophils. What is the most appropriate initial choice of antibiotic?

A. Ceftazidime
B. Clindamycin
C. Flucloxacillin
D. Tazocin

A

Answer C

Patient has features of septic arthritis.

In absence of Gram stain results, presumption is most common causative organism - Staph aureus

Treatment for MSSA - flucloxacillin

How well did you know this?
1
Not at all
2
3
4
5
Perfectly
14
Q

RACP 2021 Q18

Q18. A 17 year old boy presents with group A streptococcal positive acute rheumatic fever, with arthritis, pericarditis and a prolonged PR. What is the shortest recommended duration of antibiotics for secondary
prevention of recurrence?

A. 1 year
B. 5 years
C. 10 years
D. 15 years

A

Answer B or C ( based on interpretation of documented history)

This patient has mild cardiac involvement of rheumatic heart disease characterised by:
Echocardiogram showing:
Mild regurgitation or mild stenosis of a
single valve
OR
Atrioventricular conduction abnormality
on ECG during ARF episode

Recommendations for secondary prophylaxis:
- If documented history of ARF:
Minimum of 10 years after the most recent
episode of ARF, or until age 21 years (whichever is longer)
If NO documented history of ARF and aged <35 years - Minimum of 5 years following diagnosis of RHD or until age 21 years (whichever is longer)

https://ccmsfiles.tg.org.au/s2/PDFs/ABG16-ARF-RHD-gx-duration-prophylaxis-March-2022-update.pdf

How well did you know this?
1
Not at all
2
3
4
5
Perfectly
15
Q

RACP 2021 Q34

Q34. A man is diagnosed with Entamoeba infection after recently travelling to India. He is treated with a 10 day course of metronidazole, but also needs further luminal treatment due to which phase of the entamoeba
lifecycle?

A. Cysts
B. Merozoite
C. Schizont
D. Trophozoite

A

Answer A

  • The goals of antibiotic therapy of intestinal amebiasis are to eliminate the invading trophozoites and to eradicate intestinal carriage of the organism.
  • For all infections, regardless of the presence of symptoms - treatment with paromomycin to eliminate intraluminal cysts
  • For symptomatic infections - metronidazole or tinidazole rather than an intraluminal agent alone
  • Once treatment of invasive amoebiasis is completed, follow with a luminal agent to eradicate cysts and prevent relapse.
How well did you know this?
1
Not at all
2
3
4
5
Perfectly
16
Q

RACP 2021 Q38

Q38. A 75 year old male is diagnosed with methicillin-sensitive Staphylococcus aureus infection of his pacemaker leads. He is placed on IV flucloxacillin.
What additional treatment is required to achieve a cure?

A. Addition of gentamicin
B. Addition of probenecid
C. Addition of rifampicin and fusidic acid
D. Removal of pacemaker

A

Answer D
CIED removal (leads, including residual non-functional leads, and pulse generator recommended in the following situations:

  • Patients with TEE or other imaging demonstrating valve or lead vegetation with suspicion or confirmation of bloodstream infection
  • Patients with bacteremia due to S. aureus, coagulase-negative Staphylococcus (high grade), Cutibacterium (formerly Propionibacterium) species (high grade), Candida species, or high-grade bacteremia due to another organism with propensity to cause endocarditis
  • Patients with CIED pocket infection

Device retention may be reasonably attempted in
- absence of vegetation on TEE in the setting of bacteremia due to a gram-negative organism or S. pneumoniae
- transient bacteremia from a clear alternative source due to an organism that does not commonly cause endocarditis

How well did you know this?
1
Not at all
2
3
4
5
Perfectly
17
Q

RACP 2021 Q46

Q46. A nurse has been exposed to a needlestick injury when caring for a patient. The patient has bloods with
serology is as follows: Hepatitis B surface antigen positive, HIV serology negative and Hepatitis C serology negative. The nurse’s blood tests from an earlier in the year show: Hepatitis B core negative, HIV negative, Hepatitis C negative and Hepatitis B surface antibody > 1000 IU.
What treatment do you suggest?

A. Prophylactic Entacavir
B. Hepatitis B immunoglobulin
C. Reassure and do no investigations
D. Hepatitis surface antigen serology at 6 and 12 weeks

A

Answer C

As per eTG:
- For source Hep B surface antigen positive (or unknown) , if exposed person is immune, no further investigations or preventative treatment is needed.

This nurse has negative Hep B core ab - suggesting no previous/ current infection and has Hep B surface ab positive with titre > 1000IU. Any immunocompetent patient who has completed a course of vaccination against HBV administered according to an approved schedule is considered immune as long as their anti-HBs titre has ever been measured as being >10 mIU/mL. Hence she is immune.

How well did you know this?
1
Not at all
2
3
4
5
Perfectly
18
Q

RACP 2021 Q50

Q50. A 24 year old male with a history of IVDU presents with fevers and is found to have multiple lung abscesses.
A new tricuspid regurgitation murmur is heard and infective endocarditis is confirmed after investigations. What is
the most likely causative organism?

A. Candida albicans
B. Escherichia coli
C. Streptococcus pyogenes
D. Staphylococcus aureus

A

Answer D

S. aureus is the most common cause of right-sided IE, accounting for up to 70 percent of cases. Streptococci and enterococci are the next most common pathogens, accounting for 5 to 30 percent and 2 to 5 percent of cases, respectively. Approximately 90 percent of patients with right-sided IE are people who inject drugs (PWID).

How well did you know this?
1
Not at all
2
3
4
5
Perfectly
19
Q

RACP 2021 Q58

Q58. An 84 year old female presented to the medical ward for management of congestive cardiac failure. A routine urine MCS was done on admission. Patient has no urinary symptoms.
Urine culture: E.coli
WCC 10-100
RBC <10
Squamous epithelial cells <10
Sensitivities:
Amoxicillin sensitive
Cefuroxime parenteral sensitive
Cefuroxime oral intermediate
Trimethoprim resistant
What’s the next best step?

A. No additional intervention/management
B. Repeat urine culture
C. Oral urine alkalinating agent
D. Oral amoxicillin

A

Answer A

Patient is presenting with asymptomatic bacteriuria. Only groups that warrant treatment

  • Pregnancy
  • Due for urological procedures
  • Renal transplant recipient (< 3 months of transplant) —> eTG does not recognise this indication.
How well did you know this?
1
Not at all
2
3
4
5
Perfectly
20
Q

RACP 2021 Q65

Q65. A 70 year old female has a history of RA which is well controlled on long term methotrexate and prednisolone. She presents with a fever and shortness of breath and has audible crackles on the left lung base on auscultation. What is the most likely diagnosis?

A. Community acquired pneumonia
B. Methotrexate induced pneumonitis
C. Pulmonary embolism
D. PJP pneumonia

A
How well did you know this?
1
Not at all
2
3
4
5
Perfectly
21
Q

RACP 2021 Q69

Q69. A patient is on a vancomycin infusion for MSSA bacteremia. You are given some data:
Target concentration 20mg/L
Half life is 6 hours
Vd is 0.5L/kg
Bioavailability <5
Protein binding 30%
Patient weight 60kg
+ other information about insensible clearance
What is the maintenance dose?

A. 2mg/hr
B. 5mg/hr
C. 85mg/hr
D. 120mg/hr

A
How well did you know this?
1
Not at all
2
3
4
5
Perfectly
22
Q

RACP 2021 Q78

Q78. A patient recently returned from Thailand with fevers and was admitted to hospital. Dengue was confirmed.
Apart from bleeding, which of the following is most suggestive of severe Dengue?

A. Arthralgia
B. Ascities
C. Retro-orbital pain
D. Widespread rash

A

Answer B

Severe DENV infection includes infection with at least one of the following:
- Severe plasma leakage leading to:
* Shock
* Fluid accumulation with respiratory distress
- Severe bleeding (as evaluated by clinician)
- Severe organ involvement:
* Aspartate aminotransferase (AST) or alanine aminotransferase (ALT) ≥1000 units/L
* Impaired consciousness
* Organ failure

How well did you know this?
1
Not at all
2
3
4
5
Perfectly
23
Q

RACP 2021 Q89

Q89. A 60 year old woman with a history of bronchiectasis presents with cough productive of purulent sputum
and is commenced Amoxicillin 1g TDS with no clinical improvement. Her sputum cultures grew pan-sensitive Pseudomonas aeruginosa. How would you treat next?

A. Azithromycin
B. Ciprofloxacin
C. Inhaled tobramycin
D. Moxifloxacin

A

B ciprofloxacillin

How well did you know this?
1
Not at all
2
3
4
5
Perfectly
24
Q

RACP 2021 Q105

Q105. Persistence of which viral component prevents hepatitis B cure?

A. Covalently closed circular DNA
B. Hepatitis B surface antigen
C. Precore protein
D. Pregenomic RNA

A
How well did you know this?
1
Not at all
2
3
4
5
Perfectly
25
Q

RACP 2021 Q125

Q125. What is the most important pharmacological determinant for penicillin in terms of its bacterial eradication and clinical response?

A. AUC:MIC ratio
B. Time above MIC
C. Peak concentration
D. Trough level

A
How well did you know this?
1
Not at all
2
3
4
5
Perfectly
26
Q

RACP 2021 Q130

Q130. A patient with gastroenteritis tests positive to norovirus on stool PCR. They are admitted to hospital for management of dehydration and AKI. How long before the patient can be deisolated from contact and droplet precautions?

A. 24hrs after diarrhoea stops
B. 48hrs after diarrhoea stops
C. 72hrs after diarrhoea stops
D. Only after discharge from hospital

A

Answer B

As per CDC:

Place patients with norovirus gastroenteritis on Contact Precautions for a minimum of 48 hours after the resolution of symptoms to prevent further exposure of susceptible patients

How well did you know this?
1
Not at all
2
3
4
5
Perfectly
27
Q

RACP 2021 Q146

Q146. What drives the process of carditis in acute rheumatic fever?

A. Bacterial infection
B. Endotoxin release
C. Exotoxin release
D. Molecular mimicry

A

Answer D

Molecular mimicry implies structural similarity between some infectious or other exogenous agent and human proteins, such that antibodies and T cells activated in response to the exogenous agent react with the human protein. In ARF, antibodies directed against GAS antigens crossreact with host antigens

The alpha-helical protein structures found in M protein and N-acetyl-beta-D-glucosamine (NABG, the immunodominant carbohydrate antigen of GAS) share epitopes with myosin, and antibody crossreactivity has been demonstrated in humans

How well did you know this?
1
Not at all
2
3
4
5
Perfectly
28
Q

RACP 2021 Q153

Q153. Cereulide is the name of a toxin produced by bacillus cereus. It can lead to vomiting, most commonly how many hours after ingestion?
A. 0-6 hours
B. 7-12 hours
C. 13-18 hours
D. 19-24 hours

A

Answer A 0 -6 hrs

How well did you know this?
1
Not at all
2
3
4
5
Perfectly
29
Q

RACP 2021 Q159

Q159. Progressive multifocal leucoencephalopathy is caused by which of the following viruses?

A. JC virus
B. Adenovirus
C. EBV
D. HSV

A

Answer A

PML is a potentially fatal demyelinating disease of the central nervous system caused by a reactivation of the JC polyomavirus (JCV) that occurs almost exclusively in immunosuppressed individuals

How well did you know this?
1
Not at all
2
3
4
5
Perfectly
30
Q

RACP 2021 Q164

Q164. A patient is started in Rifampicin, Isoniazid, Pyrazinamide and Ethambutol for confirmed active TB. Which
complication of ethambutol would warrant stopping treatment with this agent?

A. Hepatitis
B. Optic neuritis
C. Adrenal failure
D. Seizures

A

Answer B

Optic neuropathy (usually manifested as a change in visual acuity or red-green colour blindness) is the most important ethambutol toxicity. The reported incidence of optic neuropathy when ethambutol is taken for more than two months increases with higher dosing of the drug. Optic neuritis is reversible in most patients. It is generally recommended that patients receiving ethambutol as part of combination therapy for treatment of a mycobacterial infection undergo baseline Snellen visual acuity and red-green colour perception testing.

How well did you know this?
1
Not at all
2
3
4
5
Perfectly
31
Q

RACP 2021 Oct Q3

  1. When does the rash occur in measles in relation to other symptoms such as
    fever, coryza, conjunctivitis?

A. At the same time as conjunctivitis
B. 2 - 4 days after fever
C. 2 - 4 days after cough
D. 1 - 2 days after coryza

A

Answer B

The exanthem of measles arises approximately two to four days after onset of fever; it consists of an erythematous, maculopapular, blanching rash, which classically begins on the face and spreads cephalocaudally and centrifugally to involve the neck, upper trunk, lower trunk, and extremities

How well did you know this?
1
Not at all
2
3
4
5
Perfectly
32
Q

RACP 2021 Oct Q73

  1. 26F at 28/40 gestation presents with fever, neck stiffness. LP high
    lymphocytes and Protein. Culture gram + cocci . Ceftriaxone is commenced and Benzylpenicillin added. What organism is the benzylpen to cover?

A. Listeria
B. H. Influenza
C. Staph aureus
D. Strep. Pneumonia

A

Answer A

Listeria monocytogenes is intrinsically resistant to cephalosporins. For patients who are older than 50 years, immunocompromised, pregnant or debilitated, or those with a history of hazardous alcohol consumption, to treat Listeria, add to the above regimen: benzylpenicillin 2.4 g (child: 60 mg/kg up to 2.4 g) intravenously, 4-hourly.

NOTE: Listeria monocytogenes are gram positive rods

How well did you know this?
1
Not at all
2
3
4
5
Perfectly
33
Q

RACP 2021 Oct Q 74

  1. 22F with 2 weeks history of behavioural change, perseveration, mutism,
    echolalia, strange neurological movements, thinks mother is someone else. What is the cause? Pt was on tramadol for joint operation.

A. Drug use
B. HSV encephalitis
C. NMDA encephalitis
D. Serotonin syndrome

A
How well did you know this?
1
Not at all
2
3
4
5
Perfectly
34
Q

RACP 2021 Oct Q79

  1. What pharmacokinetic/pharmacodynamic parameters determine efficacy
    of penicillin drugs in treatment of staph?

A. AUC:MIC ratio
B. Time above MIC.
C. Peak Level
D. ?

A
How well did you know this?
1
Not at all
2
3
4
5
Perfectly
35
Q

RACP 2021 Oct Q93

  1. What part of Hep B does Hep D need to replicate?

A. HbsAb
B. Hb e antigen
C. Core antigen

A

? None of the listed options

The hepatitis D virion comprises an RNA genome, a single HDV encoded antigen, and a lipoprotein envelope provided by hepatitis B virus (HBV). The lipoprotein envelope of HDV is provided by the HBV and consists of the same proteins (large, middle, and small S) that are found in the HB virion; their relative proportion depends upon the level of HBV replication.

How well did you know this?
1
Not at all
2
3
4
5
Perfectly
36
Q

RACP 2021 Oct Q108

  1. Pt is admitted to hospital with sepsis. Hypotensive despite fluid resus. Taken to ICU for NORAD. Pt is on 3 mg pred/day. What else can be added?

A. more fluid
B. Stress dose steroids
C. second inotrope

A
How well did you know this?
1
Not at all
2
3
4
5
Perfectly
37
Q

RACP 2021 Oct

35 yo lady ,sexually active, presents with painful knee joint. Gram stain of
aspirate shows Gm positive diplococci.

What’s the organism?
A. Chlamydia trachomatis
B. Strep pneumonia
C. Nieserria gonorrhoea
D. rhubarb

A

Answer B

Nieserria gonorrhoea - common cause of septic joint in sexually active young adults, but is Gram negative diplococci

A. Chlamydia trachomatis - Gm -ve coccobacilli
B. Strep pneumonia = Gm +ve diplococci
C. Nieserria gonorrhoea - Gm -ve diplococci
D. rhubarb

How well did you know this?
1
Not at all
2
3
4
5
Perfectly
38
Q

RACP 2020 Q1

Choose the most appropriate therapy for a 70 year old woman, with febrile neutropenia on cyclophosphamide +. Feeling well, no allergies, no hepatic or renal impairment. BP 105/80.

A. Ciprofloxacin
B. Ceftriaxone
C. Taz + Gent
D. Piperacillin-tazobactam

A

Answer C

For patients without septic shock and not requiring intensive care support (including patients with risk factors for infection with a multidrug-resistant Gram-negative bacterium, except if known to be colonised or recently infected with a resistant bacterium), use:
piperacillin+tazobactam 4+0.5 g (child: 100+12.5 mg/kg up to 4+0.5 g) intravenously, 6-hourly
OR
cefepime 2 g (child: 50 mg/kg up to 2 g) intravenously, 8-hourly [Note 4]
OR
ceftazidime 2 g (child: 50 mg/kg up to 2 g) intravenously, 8-hourly

How well did you know this?
1
Not at all
2
3
4
5
Perfectly
39
Q

RACP 2020 Q15

  1. What is the most common side effect of osteltamivir?

A) Hepatitis
B) Rash
C) Nausea and vomiting
D) Confusion

A

Answer C

Oseltamivir can cause nausea and vomiting

How well did you know this?
1
Not at all
2
3
4
5
Perfectly
40
Q

RACP 2020 Q22

  1. In addition to influenza, which vaccine is indicated in COPD?

A) Hemophilus
B) Pneumococcal
C) Varicella
D) Meningococcal

A

Answer B

How well did you know this?
1
Not at all
2
3
4
5
Perfectly
41
Q

RACP 2020a Q 36

  1. What is the most common presentation of gentamicin induced ototoxicity?

A) High frequency hearing loss
B) Low frequency hearing loss
C) Oscillopsia + vertigo
D) Ataxia + vertigo

A

Answer ? C

Gentamicin, tobramycin and streptomycin are preferentially vestibulotoxic, whereas amikacin and kanamycin are primarily cochleotoxic. Cochleotoxicity presents with hearing impairment. Vestibulotoxicity typically manifests with ataxia, dysequilibrium and oscillopsia (visual blurring with head movement), which should be elicited on history taking.1,2,6 Vertigo is usually absent, because aminoglycoside-induced toxicity causes symmetric bilateral vestibular loss

How well did you know this?
1
Not at all
2
3
4
5
Perfectly
42
Q

RACP 2020 Q29

  1. How often should patients on PrEP (pre-exposure prophylaxis) be screened for HIV?

A) 3 monthly
B) 6 monthly
C) 12 monthly
D) After every high risk exposure

A

Answer A

How well did you know this?
1
Not at all
2
3
4
5
Perfectly
43
Q

RACP 2020 Q31

  1. A 58 year old non-smoker with a past history of asthma presents to the emergency department with 4 days of productive cough and a temperature of 38.3. Her chest X-ray is shown below (pic). Which of the following is the most likely diagnosis?
    A) Empyema
    B) Middle lobe collapse/consolidation
    C) Lower lobe collapse/consolidation
    D) Pulmonary infarction

ANS: Picture was of right lower ZONE collapse consolidation. Appeared to have silhouette sign of right diaphragm but not right heart border

A
How well did you know this?
1
Not at all
2
3
4
5
Perfectly
44
Q

RACP 2020 Q32

  1. In an otherwise healthy but unvaccinated individual undergoing a splenectomy, which of the follow options is the best approach to vaccination for pneumococcus?

A) Administer conjugate vaccination before procedure then polysaccharide 8 weeks later
B) Administer polysaccharide vaccination before procedure then conjugate 8 weeks later
C) Two doses of conjugate vaccination at least 8 weeks apart
D) Two doses of polysaccharide vaccination at least 8 weeks apart

A
How well did you know this?
1
Not at all
2
3
4
5
Perfectly
45
Q

RACP 2020 Q62

  1. Sandra is 6 months post renal transplant on tacrolimus, prednisolone, and mycophenolate with increasing BK viraemia. What is an effective strategy to reduce the risk of BK nephropathy?

A) Add cidofovir
B) Prednisolone
C) Reduce tacrolimus and mycophenolate
D) Add Bactrim DS

A
How well did you know this?
1
Not at all
2
3
4
5
Perfectly
46
Q

RACP 2020 Q68

  1. Guy ate rice that was microwaved three times. Diarrhoea +++ within 3 hours. What is the likely causative organism?

A) Bacillus cereus
B) Enterotoxigenic E. coli
C) Salmonella
D) Shigella

A
How well did you know this?
1
Not at all
2
3
4
5
Perfectly
47
Q

RACP 2020 Q73

  1. What is the most important benefit of tenofovir alafenamide (TAF) compared to tenofovir disoproxil fumurate (TDF) in the treatment of human immunodeficiency virus or aquired immunodeficiency syndrome (HIV/AIDS)?

A) Blockade of chemokine receptor type 5 (CCR5)
B) Elimination of nephrotoxicity
C) Improved efficacy as pre-exposure prophylaxis (PrEP)
D) Limited potential for drug resistance

A
How well did you know this?
1
Not at all
2
3
4
5
Perfectly
48
Q

RACP 2020 Q80

  1. Needlestick injury of a nurse from a patient with HbSag positive blood. Nurse’s HbSAb titre >100IU/L

A) Hepatitis B Immunoglobulin
B) No treatment
C) Hepatitis B vaccination
D) Re-test hepatitis B antigen after 3 months

A
How well did you know this?
1
Not at all
2
3
4
5
Perfectly
49
Q

RACP 2020

  1. In addition to chorioretinitis, which ocular manifestation is associated with candidemia?

A.Endopthalmitis
B.Dacrocystitis
C. Iritis
D. Keratitis

A
How well did you know this?
1
Not at all
2
3
4
5
Perfectly
50
Q

RACP 2020

  1. What is the vaccination regime for pneumococcus in a patient with a splenectomy?

A) Conjugate vaccine and repeat conjugate vaccine in 8 weeks
B) Conjugate vaccine first and polysaccharide vaccine in 8 weeks
C) Polysaccharide vaccine and repeat polysaccharide vaccine in 8 weeks
D) Polysaccharide vaccine first and conjugate vaccine 8 weeks after

A
How well did you know this?
1
Not at all
2
3
4
5
Perfectly
51
Q

RACP 2020

  1. Known asthmatic. Febrile. SOB. Radiograph as shown - what is the diagnosis?

A) Right Middle lobe collapse/consolidation
B) Right lower lobe collapse/consolidation
C) Sarcoidosis

A
How well did you know this?
1
Not at all
2
3
4
5
Perfectly
52
Q

RACP 2020

  1. A 27 year old patient from Zimbabwe who migrated to Australia when he was 19 presents with his first episode of a seizure. No other background medical history. The MRI shows calcific nodules (see picture).

A. Cryptococcus
B. HIV encephalopathy
C. Toxoplasmosis
D. Neurocysticercosis

A
How well did you know this?
1
Not at all
2
3
4
5
Perfectly
53
Q

RACP 2020

  1. A patient with a prosthetic aortic valve is undergoing a dental extraction. What prophylaxis should he receive?

A. Amoxicillin monotherapy
B. Amoxicillin-Clavulanic Acid
C. No prophylaxis
D. Amoxicillin + Metronidazole

A
How well did you know this?
1
Not at all
2
3
4
5
Perfectly
54
Q

RACP 2020

  1. Recurrent MSSA boils & infections. When should mupirocin 2% decolonisation be performed?
A
How well did you know this?
1
Not at all
2
3
4
5
Perfectly
55
Q
  1. Young non-pregnant woman with a symptomatic UTI (dysuria, urinary frequency). No flank pain. Afebrile. Urine culture shows 300 WCC and fully sensitive E.Coli. What treatment should be initiated?

A) Ciprofloxacin
B) Gentamicin
C) Ceftriaxone
D) Trimethoprim

A
How well did you know this?
1
Not at all
2
3
4
5
Perfectly
56
Q

RACP 2020

  1. Man in his 30s who works with stone kitchen bench tops presenting with dry cough. X-ray shows bilateral upper fibrosis. What is the diagnosis?

A) Tuberculosis
B) Silicosis
C) Asbestosis
D) Hypersensitivity pneumonitis

A
How well did you know this?
1
Not at all
2
3
4
5
Perfectly
57
Q

RACP 2020

  1. Which antibiotic is most associated with QT prolongation?

A) Ceftriaxone
B) Amoxicillin
C) Gentamicin
D) Ciprofloxacin

A
How well did you know this?
1
Not at all
2
3
4
5
Perfectly
58
Q

RACP 2020

  1. What is the most common mechanism of carbapenem resistance in gram negative organisms?

A) Efflux pumps
B) Drug hydrolysis
C) Beta-lactamases

A
How well did you know this?
1
Not at all
2
3
4
5
Perfectly
59
Q

RACP 2020

  1. A patient at high risk of HIV on PrEP. How often should HIV testing be performed?

A. After every sexual encounter
B. 3 monthly
C. 6 monthly
D. Yearly

A
How well did you know this?
1
Not at all
2
3
4
5
Perfectly
60
Q

RACP 2019b Q5

Question 5
Treatment of asymptomatic bacteriuria is recommended before which procedure?

A. CABG
B. TURP
C. Total hip replacement
D. Colonoscopy

A
How well did you know this?
1
Not at all
2
3
4
5
Perfectly
61
Q

RACP 2019 Q1

What is the response rate of ledipasvir + sofosbuvir in Hepatitis C genotype 1?

A. 65%
B. 75%
C. 85%
D. 95%

A
How well did you know this?
1
Not at all
2
3
4
5
Perfectly
62
Q

RACP 2019 Q35

A patient with replapsing-remitting multiple sclerosis has been treated with natalizumab for 4 years. She develops new neurological symptoms and MRI shows white matter changes. Progressive multifocal leukoencephalopathy is suspected. What is the most specific CSF test?

A. Neuron specific enolase
B. JC virus
C. Oligoclonal bands
D. 14-3-3 protein

A
How well did you know this?
1
Not at all
2
3
4
5
Perfectly
63
Q

RACP 2019 Q45

What vaccination besides H. influzenzae, pneumoccoal and meningococcal would you give to an asplenic patient?

A. Bordetella
B. Rubella
C. Influenza
D. Varicella zoster

A
How well did you know this?
1
Not at all
2
3
4
5
Perfectly
64
Q

RACP 2019 Q46

A 30 year old man presents with a three month history of bloody diarrhoea associated with mucous, tenesmus and three kilograms of weight loss. He last travelled to Thailand six months ago. Blood tests revealed an anaemia with Hb 115 g/L, CRP 80 mg/L, ESR 25 mm/Hr. WCC, B12, iron and folate were normal. What is the most likely diagnosis?

A. Crohn’s disease
B. Ulcerative Colitis
C. Ischaemic colitis
D. Salmonella enteritis

A
How well did you know this?
1
Not at all
2
3
4
5
Perfectly
65
Q

RACP 2019 Q54

A 35 year old man presents with progressive cough for more than a year. He works in an office, does not smoke, lives alone, and has not travelled overseas. There is no family history of lung disease and he reports no contact with other persons with pulmonary disease or chest infection. Clinical examination is unremarkable. His chest x-ray is shown:

?Unexciting CXR – not the same as last year’s though
Which of the following is the most likely diagnosis?

A. Extrinsic allergic alveolitis
B. Lung cancer
C. Sarcoidosis
D. Tuberculosis

A
How well did you know this?
1
Not at all
2
3
4
5
Perfectly
66
Q

RACP 2019 Q57

A 25 year old man is admitted with severe necrotising fasciitis. He has grown Streptococcus pyogenes in his blood culture and is started on high dose IV benzylpenicillin. Which of the following antimicrobials would you add?

A. Ceftriaxone
B. Clindamycin
C. Moxifloxacin
D. Vancomycin

A

Answer B

Clindamycin or lincomycin is recommended for S. pyogenes necrotising fasciitis because of a theoretical reduction in bacterial toxin production; however, clinical evidence is limited.

How well did you know this?
1
Not at all
2
3
4
5
Perfectly
67
Q

RACP 2019 Q77

A 54 year old man has stage 3 colorectal cancer on capcitabine and oxaliplatin. He is currently experiencing 8 episodes of diarrhoea a day, despite taking loperamide. There are no other infective symptoms. What medication would you add to manage his symptoms?

A. Methylprednisolone
B. Subcutaneous morphine
C. Metronidazole
D. Infliximab

A
How well did you know this?
1
Not at all
2
3
4
5
Perfectly
68
Q

RACP 2019 Q89

A 26yo Indigenous woman presents with arthralgias, rash and a new cardiac murmur on auscultation. She mentions she had been unwell recently with a sore throat. What is the most likely diagnosis?

A. Acute rheumatic fever
B. Infective endocarditis
C. Sarcoidosis
D. Pericardial effusion

A

Answer A (ARF)

How well did you know this?
1
Not at all
2
3
4
5
Perfectly
69
Q

RACP 2019 P2 Q6

Patient returns from India with a UTI and on subsequent urinalysis found to isolate ESBL.
What is the most likely mechanism of antibiotic resistance?

A. Drug efflux
B. Drug Hydrolysis
C. Altered binding protein
D. Decreased permeability

A
How well did you know this?
1
Not at all
2
3
4
5
Perfectly
70
Q

RACP 2019 P2 Q12

In patients with malaria treated with artemether and lumefantrine, they are then given two weeks of primaquine. Which part of the malaria life cycle does primaquine treat?

A. Blood schizonts
B. Hypnozoites
C. Trophozoites
D. Sporozoites

A
How well did you know this?
1
Not at all
2
3
4
5
Perfectly
71
Q

RACP 2019 P2 Q34

There is an outbreak of vomiting and diarrhoea in a nursing home. The vomiting and diarrhoea lasts for approximately 24 hours before resolving but spreads rapidly throughout residents. What is the most likely pathogen?

A. Enteric adenovirus
B. Rotavirus
C. Norovirus
D. Coronavirus

A
How well did you know this?
1
Not at all
2
3
4
5
Perfectly
72
Q

RACP 2019 P2 Q55

A 19 year old female is admitted with meningococcal meningitis. What contact precautions are required in the first 24 hours of her admission?

A. Negative pressure
B. Droplet
C. Airborne
D. Contact

A
How well did you know this?
1
Not at all
2
3
4
5
Perfectly
73
Q

RACP 2019 P2 EMQ (incomplete)

Q: Autoinfection and hyperinfection in immunocompromised

A. CMV
B. Pneumocytis jirovecii
C. Strongyloides
D. Burkholderia

A
How well did you know this?
1
Not at all
2
3
4
5
Perfectly
74
Q

RACP 2019 P2 EMQ (incomplete)

Q. Transplant patients need prophylaxis regardless of donor or recipient status

A. CMV
B. Pneumocytis jirovecii
C. Strongyloides
D. Burkholderia

A
How well did you know this?
1
Not at all
2
3
4
5
Perfectly
75
Q

RACP 2018 Q17

A 76-year-old man with prior mechanical aortic valve replacement requires a colonoscopy and likely colonic biopsy. What endocarditis prophylaxis is appropriate?

A. Ampicillin.
B. Ceftriaxone.
C. Gentamicin.
D. Metronidazole.
E. No prophylaxis.

A

Answer E - No prophylaxis

Antibiotic prophylaxis for genitourinary and gastrointestinal tract procedures is indicated only if surgical antibiotic prophylaxis is required or for patients with an established infection

How well did you know this?
1
Not at all
2
3
4
5
Perfectly
76
Q

RACP 2018 Q25

  1. A patient presents with severe sepsis, hypotension and a swollen red area of cellulitis on the right leg. You suspect necrotising fasciitis. Confirmation of the diagnosis is best made by which of the following?

A. Clinical crepitus.
B. Clinical response to IV antibiotics.
C. CT imaging.
D. Culture of group A Streptococcus.
E. Surgical exploration.

A

Answer E

Definitive diagnosis of necrotising fasciitis is made on visualisation of the tissue on surgical exploration.

CT imaging with contrast can show enhancement at areas of necrosis but should not delay surgical debridement.

Clinical crepitus is suggestive of gas in soft tissue and is present in 50% of patients with NF.

How well did you know this?
1
Not at all
2
3
4
5
Perfectly
77
Q

RACP 2018a Q60

  1. Which factor is the strongest indication for surgery in native valve bacterial endocarditis?

A. Intravenous drug use and tricuspid valve endocarditis.
B. Mobile vegetation of 5 mm on echocardiography.
C. Pulmonary infarction.
D. Staphylococcal bacteraemia for 3 days despite therapy.
E. Valve regurgitation resulting in heart failure.

A

Answer E

Indications for surgery in IE:
- Prosthetic valve endocarditis
- Valve dysfunction leading to heart failure
- Uncontrolled infection: e.g., enlarging vegetation, persistent bacteremia (> 7 days of appropriate treatment)
- Perivalvular extension or complications: e.g., abscess, pseudoaneurysm, fistula, heart block
- Fungal endocarditis
- High embolic risk: e.g., mobile vegetation > 10 mm, recurrent embolism

How well did you know this?
1
Not at all
2
3
4
5
Perfectly
78
Q

RACP 2018 Q61

A 40-year-old HIV positive man with a CD4 count of 450 cells/μL [650–2000] presents with a 3 day history of fever, cough, sputum production and a right lower lobe consolidation on chest x-ray. What is the most likely organism causing his pneumonia?

A. Methicillin resistant Staphylococcus aureus.
B. Mycobacterium avium complex.
C. Mycobacterium tuberculosis.
D. Pneumocystis jirovecii.
E. Streptococcus pneumoniae.

A
How well did you know this?
1
Not at all
2
3
4
5
Perfectly
79
Q

RACP 2018 Q77

A 32-year-old man has returned from 8 weeks in India 16 days previously. Since his arrival home, he has had fevers, abdominal pain, constipation and lethargy. Gram negative bacilli have grown in both aerobic and anaerobic blood culture samples 13 hours after being drawn. What is the most likely diagnosis?

A. Dengue.
B. Escherichia coli O157.
C. Leptospirosis.
D. Malaria.
E. Typhoid.

A
How well did you know this?
1
Not at all
2
3
4
5
Perfectly
80
Q

RACP 2018 Q78

A 90-year-old resident of a nursing home develops offensive diarrhoea after a 7-day period of constipation. He is being treated with paracetamol and a transdermal opioid for chronic persistent pain and has also received long-term coloxyl with senna two tablets daily. He had a 5-day course of cephalexin for a urinary tract infection 5 days ago. What is the most likely cause for the diarrhoea?

A. Clostridium difficile.
B. Diverticulitis.
C. Drug-induced colitis.
D. Laxative abuse.
E. Overflow diarrhoea.

A
How well did you know this?
1
Not at all
2
3
4
5
Perfectly
81
Q

RACP 2018 Q80

A 35-year-old man presents with fevers and myalgias 10 days after returning to Australia from Uganda. He had no pre-travel advice or vaccinations and did not take antimalarials.
Investigations reveal Plasmodium falciparum (parasite count 9.2%), anaemia (haemoglobin
85 g/L [135–175]), thrombocytopoenia (platelets 40 × 109/L [150–400]), acute kidney injury and a raised bilirubin (60 μmol/L [3–18]). The hospital has all antimalarial therapies available for emergency use.
Which antimalarial agent should be commenced?

A. Intravenous artesunate monotherapy.
B. Intravenous quinine dihydrochloride monotherapy.
C. Oral combination artemether+lumefantrine.
D. Oral doxycycline monotherapy.
E. Oral mefloquine monotherapy.

A
How well did you know this?
1
Not at all
2
3
4
5
Perfectly
82
Q

RACP 2018 Q81

  1. Use of antibiotics (especially vancomycin and broad-spectrum antibiotics) leads to an increase in the incidence of vancomycin-resistant Enterococcus (VRE).
    What other factor contributes to colonisation with VRE?

A. Antifungal drugs.
B. Chronic immunosuppression.
C. Clostridium difficile infection.
D. Diabetes mellitus.
E. Residential care.

A
How well did you know this?
1
Not at all
2
3
4
5
Perfectly
83
Q

RACP 2018 Q97

  1. Which antifungal is first-line therapy for treatment of invasive aspergillosis?

A. 5-Flucytosine.
B. Caspofungin.
C. Fluconazole.
D. Griseofulvin.
E. Itraconazole.
F. Liposomal amphotericin.
G. Terbinafine.
H. Voriconazole.

A
How well did you know this?
1
Not at all
2
3
4
5
Perfectly
84
Q

RACP 2018 Q98

  1. Which antifungal is first-line therapy for an intensive care patient with Candida glabrata candidaemia?
    A. 5-Flucytosine.
    B. Caspofungin.
    C. Fluconazole.
    D. Griseofulvin.
    E. Itraconazole.
    F. Liposomal amphotericin.
    G. Terbinafine.
    H. Voriconazole.
A
How well did you know this?
1
Not at all
2
3
4
5
Perfectly
85
Q

RACP 2018 Q 106

106.The in vitro gamma interferon release assay Quantiferon TB GoldTM is commonly used in patients with suspected Mycobacterium tuberculosis (MTB).
What is the main role of the Quantiferon Gold test?

A. Detect exposure to MTB.
B. Determine extent of MTB disease.
C. Determine response to MTB therapy.
D. Predict active MTB.
E. Rule out MTB.

A
How well did you know this?
1
Not at all
2
3
4
5
Perfectly
86
Q

RACP 2018 Q109

109.Which of the following is a minor criterion for the diagnosis of acute rheumatic fever?

A. Carditis.
B. Polyarthritis.
C. Prolonged PR interval.
D. Proteinuria.
E. Subcutaneous nodules.

A
How well did you know this?
1
Not at all
2
3
4
5
Perfectly
87
Q

RACP 2018 Q117

117.Molecular mimicry may account for the development of carditis in acute rheumatic fever. The streptococcal M protein and N-acetyl-beta-D-glucosamine share epitopes with which component of cardiac tissue?

A. Actin.
B. Desmosomes.
C. Myosin.
D. T-tubules.
E. Troponin.

A
How well did you know this?
1
Not at all
2
3
4
5
Perfectly
88
Q

RACP 2018 Q145

145.Kidney stones contain a variety of crystalline and non-crystalline material. Stone composition can influence urological intervention. Which crystalline material is associated with Proteus mirabilis, a urease-producing bacteria?

A. Calcium oxalate.
B. Calcium phosphate.
C. Cystine.
D. Struvite.
E. Uric acid.

A
How well did you know this?
1
Not at all
2
3
4
5
Perfectly
89
Q

RACP 2018b Q170

170.Which antibiotic is most commonly associated with this adverse drug reaction?
Pulmonary fibrosis.

A. Azithromycin.
B. Cefaclor.
C. Cefepime.
D. Ciprofloxacin.
E. Clindamycin.
F. Flucloxacillin.
G. Nitrofurantoin.
H. Rifampicin.

A
How well did you know this?
1
Not at all
2
3
4
5
Perfectly
90
Q

RACP 2018 Q169

Which antibiotic is most commonly associated with this adverse drug reaction?

169.Achilles tendonitis and rupture.
A. Azithromycin.
B. Cefaclor.
C. Cefepime.
D. Ciprofloxacin.
E. Clindamycin.
F. Flucloxacillin.
G. Nitrofurantoin.
H. Rifampicin.

A
How well did you know this?
1
Not at all
2
3
4
5
Perfectly
91
Q

RACP 2018 Q164

164.Vibrio cholerae causes diarrhoea primarily by what mechanism?

A. Hypermotility.
B. Inflammatory.
C. Malabsorption.
D. Osmotic.
E. Secretory.

A
How well did you know this?
1
Not at all
2
3
4
5
Perfectly
92
Q

RACP 2017

A 75-year-old Chinese man who is being treated with a TNF inhibitor for Rheumatoid Arthritis develops fevers, night sweats and a productive cough. You suspect Tuberculosis.  The most sensitive test for diagnosing active TB infection is:

A. Three consecutive sputum samples sent for acid fast bacilli staining
B. Three consecutive sputum samples sent for TB culture
C. Transbronchial biopsy
D. Quantiferon Gold Assay
E. Tuberculin Skin Test

A
How well did you know this?
1
Not at all
2
3
4
5
Perfectly
93
Q

RACP 2017

A 31-year-old male has had a renal transplant five years ago, and he presents to you with a headache, fever and signs of meningism.
The salient features of the LP are:
Mild elevation of protein
Low Glucose
WCC 700 (Neutrophils 630 Mononuclear 70)
RBC 52
Gram stain and PCR is pending
What is the most likely causative agent?

A. Herpes Simplex
B. Listeria monocytogenes
C. Streptococcus pneumonia
D. Klebsiella pneumonia
E. Mycobacterium tuberculosis

A
How well did you know this?
1
Not at all
2
3
4
5
Perfectly
94
Q

RACP 2017

A 35-year-old gentleman presents with poorly demarcated erythema, pain, and tenderness involving the skin around his left eye. You notice there is a degree of chemosis involving the left eye. The patient states that movement of his left eyeball is restricted and painful.Which of the following treatments is most appropriate for managing this condition?
What is the most appropriate empirical antibiotic regimen?

A. Clindamycin
B. Benzylpenicillin and Metronidazole
C. Flucloxacillin and Ceftriaxone
D. Flucloxacillin
E. Ciprofloxacin

A
How well did you know this?
1
Not at all
2
3
4
5
Perfectly
95
Q

RACP 2017

An 82-year-old female with hypertension, Type 2 Diabetes and Chronic Kidney Disease (eGFR 35 mL/min/1.73m2) presents to the outpatient clinic with an uncomplicated urinary tract infection. She’s current on Spironolacto’e, Losartan and Gliclazide. Which of the following is the most appropriate antibiotic agent?

A. Cephalexin
B. Ciprofloxacin
C. Nitrofurantoin
D. Norfloxacin
E. Trimethoprim

A
How well did you know this?
1
Not at all
2
3
4
5
Perfectly
96
Q

RACP 2017

A 25-year-old HIV negative man who has sex with other men presented with a fever, rash and malaise. His Treponema Pallidum Particle Agglutination assay is positive and he has a Rapid Plasma Reagin titre of 1:64. At that time he was diagnosed with secondary Syphilis infection and treated with one dose of IM Benzathine Penicillin.
One year later, his RPR titre is noted to be 1:256. What is the most likely explanation for his markedly elevated RPR titre?

A. Co-infection with Chlamydia trachomatis
B. Re-infection with Syphilis
C. Insufficient initial treatment
D. Antibiotic resistance
E. False positive result

A
How well did you know this?
1
Not at all
2
3
4
5
Perfectly
97
Q

RACP 2017

A 35-year-old gentleman presents with abdominal pain and fevers after returning from a three-month holiday in India. Abdominal CT was performed which shows a liver abscess. The patient is commenced on Ceftriaxone and Metronidazole. What is the most likely organism?

A. Aeromonas veronii
B. Entamoeba histolytica
C. Salmonella typhi
D. Shigella dysenteriae
E. Escherichia Coli

A
How well did you know this?
1
Not at all
2
3
4
5
Perfectly
98
Q

RACP 2017

A 28-year-old man presents with fever, neck stiffness and photophobia after spending one week in Thailand. A lumbar puncture is consistent with eosinophilic meningitis.
What is the most likely causative organism?

A. Ascaris lumbricoides
B. Angiostrongylus cantonensis
C. Enterobius vermicularis
D. Schistosoma haematobium
E. Taenia saginata

A
How well did you know this?
1
Not at all
2
3
4
5
Perfectly
99
Q

RACP 2016 Q48

A 25 year old healthy male returns from Papua New Guinea with fevers. He is systemically apart from a fever. Investigations in ED confirm a diagnosis of malaria but identification of the type is not available until the morning. What is the most appropriate treatment to start?

A. Lumefantrine and arthmeter
B. Artesunate
C. Chloroquine
D. Mefloquine
E. Quinine

A
How well did you know this?
1
Not at all
2
3
4
5
Perfectly
100
Q

RACP 2016 Q60

A 25 year old male is diagnosed with MSSA endocarditis with a perivalvular vegetation. He is commenced on IV flucloxacillin. What is the optimal duration of treatment?

A. 2 weeks
B. 4 weeks
C. 6 weeks
D. 8 weeks
E. 12 weeks

A

Answer C - 6 weeks

For patients with uncomplicated native valve endocarditis caused by methicillin-susceptible staphylococci, 4 weeks of antibiotic therapy is often adequate. For patients with complicated infections (eg perivalvular abscess, septic metastatic complications), use 6 weeks of therapy.

How well did you know this?
1
Not at all
2
3
4
5
Perfectly
101
Q

RACP 2016 Q65

A 32 year old man with treated HIV presents with painless ulcer on the glans penis with curled edge. He report he is still having unprotected sex.
What is the most likely cause of the legion?

A. Chancroid
B. Donovanosis
C. Herpes simplex
D. Lupus vulgaris
E. Syphilis

A
How well did you know this?
1
Not at all
2
3
4
5
Perfectly
102
Q

RACP 2016 Q81

A 60year old presents febrile and neutropenic post induction chemotherapy for AML. A CT chest is concerning for fungal infection and bronchoscopy culture demonstrates Aspergillus fumigatus.
What is the most appropriate treatment?

A. Posaconazole
B. Fluconazole
C. Itraconazole
D. Carbimazole
E. Voriconazole

A
How well did you know this?
1
Not at all
2
3
4
5
Perfectly
103
Q

RACP 2016 Q97 EMQ

Question 97
Bloody diarrhoea in a traveller one day back from Thailand

A. Cryptosporidium
B. Yersinia
C. Rotavirus
D. Shigella
E. Enterotoxigenic E. Coli (ETEC)
F. Toxoplasmosis
G. Vibrio Cholerae
H. Tuberculosis

A
How well did you know this?
1
Not at all
2
3
4
5
Perfectly
104
Q

RACP 2016 Q98 EMQ

The most likely cause of chronic watery diarrhoea in a HIV patient with CD4 count < 200.

A. Cryptosporidium
B. Yersinia
C. Rotavirus
D. Shigella
E. Enterotoxigenic E. Coli (ETEC)
F. Toxoplasmosis
G. Vibrio Cholerae
H. Tuberculosis

A
How well did you know this?
1
Not at all
2
3
4
5
Perfectly
105
Q

RACP 2016 P2 Q16

A urinalysis was positive for leukocytes but negative for nitrites. A microorganism was grown on culture. Which microorganism is it most likely? 

A. Pseudomonas aeruginosa
B. Enterococcus faecalis
C. E.coli
D. Enterobacter cloacae
E. Klebsiella pneumoniae

A
How well did you know this?
1
Not at all
2
3
4
5
Perfectly
106
Q

RACP 2016 P2 Q38

  1. What is the optimal timing of cephazolin administration for a surgical joint replacement?

A. 12 hours pre and post surgery
B. 24 hours pre and 48 hours post surgery
C. 24 hours pre surgery
D. At time of incision
E. 60 minutes prior to surgery

A

Answer E

How well did you know this?
1
Not at all
2
3
4
5
Perfectly
107
Q

RACP 2016 P2 Q57

Question 57
Haemolytic Uraemic Syndrome and bloody diarrhoea can occur as a consequence of EHEC. The pathogenesis involves the production of which of the following?

A. Shiga toxin
B. Heat stable endotoxin
C. Intimin
D. Cytotoxin
E. Adherin

A
How well did you know this?
1
Not at all
2
3
4
5
Perfectly
108
Q

RACP 2015 Q10

Clindamycin is often added to meropenem to treat a suspected necrotising fasciitis. The reason for this is:

A. To enhance the activity of meropenam
B. To treat MRSA
C. For anaerobic coverage
D. To cover Streptococcus pyogenes toxin
E. To overcome resistance to meropenem

A

Answer D

Clindamycin or lincomycin is recommended for S. pyogenes necrotising fasciitis because of a theoretical reduction in bacterial toxin production; however, clinical evidence is limited.

How well did you know this?
1
Not at all
2
3
4
5
Perfectly
109
Q

RACP 2015 Q29

What is the most common side effect of the antimicrobial therapy for Enterococcal endocarditis?

A. Neuropathy
B. Encephalopathy
C. Aplastic anaemia
D. Vestibular toxicity

A

Answer D Vestibular toxicity

Enterococci are intrinsically more resistant to bactericidal antibiotics than other common causes of infective endocarditis, so require a synergistic combination of bactericidal antibiotics for a prolonged duration (usually 6 weeks). Therefore, even if the pathogen is susceptible to penicillin or vancomycin, treat enterococcal endocarditis concomitantly with gentamicin (unless there is high-level gentamicin resistance, or gentamicin is contraindicated or relevant precautions preclude its use).
Treatment for Enterococcal IE
IV benzylpenicillin OR IV ampicillin OR IV amoxicillin PLUS gentamicin

Common side effects of gentamicin - nephrotoxicity and ototoxicity; uncommon - neuromuscular blockade, peripheral neuropathy and bronchospasm

How well did you know this?
1
Not at all
2
3
4
5
Perfectly
110
Q

RACP 2015 Q39

A 25 year old man presents to hospital two days after returning from Fiji with fever, rash and arthralgia’s. Dengue fever is confirmed on PCR and antigen testing. What signifies his risk of severe dengue and shock?

A. Decreased lymphocyte count
B. Increased creatinine
C. Increases haematocrit
D. Worsening diarrhoea
E. Worsening rash

A
How well did you know this?
1
Not at all
2
3
4
5
Perfectly
111
Q

RACP 2015 Q46

What would be the most likely serology finding in a patient with acute EBV infection:

A
How well did you know this?
1
Not at all
2
3
4
5
Perfectly
112
Q

RACP 2015 Q62

What is the cause of increased shock in sepsis?

A. Increased preload
B. Decreased cardiac output
C. Decreased heart rate
D. Decreased peripheral vascular resistance
E. Increased afterload

A
How well did you know this?
1
Not at all
2
3
4
5
Perfectly
113
Q

RACP 2015 Q48

What is the most common manifestation of secondary immunodeficiency caused by chronic systemic steroid use?

A. Bacterial bronchopneumonia
B. Folliculitis
C. Herpes zoster
D. Oral candidiasis
E. Viral meningitis

A
How well did you know this?
1
Not at all
2
3
4
5
Perfectly
114
Q

RACP 2015 Q57

Which of the following populations would you treat in asymptmatic bacteriuria

A. Nursing home patient
B. Spinal cord injury with IDC
C. Pregnant woman 28 weeks
D. General medical patient
E. Someone about to have hip/knee replacement

A
How well did you know this?
1
Not at all
2
3
4
5
Perfectly
115
Q

RACP 2015

A 45 year old woman is diagnosed with endocarditis involving her native mitral valve. Blood cultures yield Enterococcus faecalis demonstrating sensitivity to ampicillin and vancomycin, but high-level resistance to gentamicin. What antibiotic regimen should she receive?

A. Ampicillin monotherapy
B. Vancomycin monotherapy
C. Ampicillin and vancomycin
D. Ampicillin and gentamicin
E. Ampicillin and ceftriaxone

A

Answer E Ampicillin and ceftriaxone

For patients with native or prosthetic valve E. faecalis endocarditis susceptible to penicillin, but with either high-level aminoglycoside resistance, or relevant contraindications or precautions that preclude the use of gentamicin as a two-drug regimen, use:
- Amoxicillin 2 g intravenously, 4-hourly for 6 weeks OR
- Ampicillin 2 g intravenously, 4-hourly for 6 weeks PLUS (with either of the above drugs)
- Ceftriaxone 2 g intravenously, 12-hourly for 6 weeks

How well did you know this?
1
Not at all
2
3
4
5
Perfectly
116
Q

RACP 2015 Q66

58 M with mitral valve endocarditis, blood cultures grow Enterococcus faecalis that is fully sensitive to penicillin as well as vancomycin. In addition to amoxicillin which medication would you use intravenously to treat his endocarditis?

A. Ceftriaxone
B. Doxycycline
C. Linezolid
D. Quinupristin and dalfopristin
E. Teicoplanin

A

Answer A

Enterococci are intrinsically more resistant to bactericidal antibiotics than other common causes of infective endocarditis, so require a synergistic combination of bactericidal antibiotics for a prolonged duration (usually 6 weeks). Therefore, even if the pathogen is susceptible to penicillin or vancomycin, treat enterococcal endocarditis concomitantly with gentamicin (unless there is high-level gentamicin resistance, or gentamicin is contraindicated or relevant precautions preclude its use - then replace gentamicin with ceftriaxone).

How well did you know this?
1
Not at all
2
3
4
5
Perfectly
117
Q

RACP 2014

80 y.o male presents tot the outpatient department with bilateral leg erythema. He has a normal white cell count and CRP. Swabs are taken which grow staph aureus, sensitive to flucloxacilin, and pseudomonas aeruginosa sensitive to ceftazidime and timentin. What is the next best management?

A. Flucloxacillin and clindamycin
B. Timentin
C. Topical steroid cream
D. Compression bandages
E. Topical emollients.

A
How well did you know this?
1
Not at all
2
3
4
5
Perfectly
118
Q

RACP 2014 Q 26

A middle aged female has returned from a holiday in India with urinary symptoms. A urine culture reveals a gram-negative bacteria resistant to ampicillin, vancomycin and meropenem. What is the most appropriate choice of intravenous antibiotic?

a) Colistin
b) Linezolid
c) Tigecycline
d) Moxifloxacin
e) Irtapenem

A
How well did you know this?
1
Not at all
2
3
4
5
Perfectly
119
Q

RACP 2014 Q47

A 20 year old male presents with a 3 day history of cough, fever, cold and conjunctivitis. He has a maculopapular rash.

Which diagnosis would require urgent isolation?

A. Influenza A
B. EBV
C. Measles
D. Rubella
E. Syphilis

A
How well did you know this?
1
Not at all
2
3
4
5
Perfectly
120
Q

RACP 2014 Q7

A 32 year old woman presents with abdominal pain, bloody diarrhoea and fever. Two days earlier she ate takeaway chicken. Her husband shared this meal with her, and is similarly unwell. They have had no recent overseas travel.

What is the most likely causative organism?

a. Cytomegalovirus
b. Norovirus
c. Staphylococcus aureus
d. Campylobacter jejuni
e. Bacillus cereus

A
How well did you know this?
1
Not at all
2
3
4
5
Perfectly
121
Q

RACP 2014 Q37

Urine dipstick nitrites is a useful tool in the ED for helping to diagnose urinary tract infections. Which of these organisms is likely to be negative for dipstick nitrites in the setting of culture-proven UTI?

A. Enterobacter cloacae
B. Enterococcus faecalis
C. Escherichia coli
D. Klebsiella oxytoca
E. Pseudomonas aeruginosa

A
How well did you know this?
1
Not at all
2
3
4
5
Perfectly
122
Q

RACP 2013 Q6

A patient with a history of psychosis and depression (who is on on medication for both) is brought in to ED unconscious, febrile at 40 degrees, tachycardic and with muscle rigidity. What is the most likely diagnosis?

A. Serotonin syndrome
B. Malignant neuroleptic syndrome
C. Acute dystonia
D. Sepsis

A
How well did you know this?
1
Not at all
2
3
4
5
Perfectly
123
Q

RACP 2013 Q7

In a diabetic patient with foot ulcers, which sign best predicts osteomyelitis?

A. Ability to probe to bone
B. Extensive cellulitis
C. Local pain
D. Malodorous discharge
E. Capillary refill >2 sec

A

Answer A

  • A positive probe-to-bone test is strongly suggestive of osteomyelitis, especially in diabetic patients with risk factors for osteomyelitis.
  • Probing to bone with a sterile blunt metal tool should be included in the initial assessment of diabetic patients with infected pedal ulcers. A positive result consists of detection of a hard, gritty surface.
  • In a systematic review evaluating the performance of the probe-to-bone test (using bone histopathology or culture as the reference standard), the pooled sensitivity and specificity for the test were 87 and 83 percent, respectively.
How well did you know this?
1
Not at all
2
3
4
5
Perfectly
124
Q

RACP 2013 Q36

A 75 y.o female presents with a vesicular rash suggestive of VZV. In what nerve involvement would the presentation mimic meningitis.

a) Trigeminal nerve (mandibular branch)
b) Trigeminal nerve (ophthalmic branch)
c) Facial nerve
d) Glossopharyngeal nerve
e) Greater occipital nerve

A
How well did you know this?
1
Not at all
2
3
4
5
Perfectly
125
Q

RACP 2013 Q54

Question 54
What presentation of invasive Neisseria species infection has the highest mortality?

a) Gonococcal infection
b) Pneumonia
c) Meningococcaemia
d) Meningitis
e) Disseminated gonococcal infection

A
How well did you know this?
1
Not at all
2
3
4
5
Perfectly
126
Q

RACP 2013 Q67

Question 67
A 67yo male from the community presents with subacute onset clubbing, haematuria, and a new mitral regurgitation murmur

A. Aspergillus niger
B. E. Coli
C. Klebsiella pneumoniae
D. Enterococcus faecalis
E. Staph aureus
F. Staph mitis

A
How well did you know this?
1
Not at all
2
3
4
5
Perfectly
127
Q

RACP 2013 Q68

Question 68
A 50yo patient in ICU with pneumonia with a femoral central line in situ and currently on Tazocin is noted to have a new fever and drop in BP

A. Aspergillus niger
B. E. Coli
C. Klebsiella pneumoniae
D. Enterococcus faecalis
E. Staph aureus
F. Staph mitis

A
How well did you know this?
1
Not at all
2
3
4
5
Perfectly
128
Q

RACP 2013 P2 Q4

Question 4
A 30 year old gentleman 200 days post cord stem cell transplant presents with fever and feeling slightly unwell. His BP is 110/70mmHg and has a heart rate of 100bpm. He has widespread lesions on his skin as shown below

Where should he be best managed?
A. HDU
B. Negative pressure room
C. Positive pressure room
D. Ward
E. Home

A
How well did you know this?
1
Not at all
2
3
4
5
Perfectly
129
Q

RACP 2013 P2 Q29

A 48F who has recently received 6 cycles of multi-agent chemotherapy for non-Hodgkin lymphoma presents with a 2 week history of diplopia and bladder disturbance, and a 3 day history of leg weakness. On examination, she has a left 6th cranial nerve palsy, weakness in the L5 distribution of the L leg, and tendon reflexes which are difficult to elicit. A lumbar puncture is performed, which demonstrates elevated protein and WBC (90% lymphocytes). What is the most likely diagnosis?

A. CMV infection
B. Paraneoplastic phenomenon
C. Lymphoma recurrence
D. Cryptococcal meningitis
E. Cerebral tuberculosis

A
How well did you know this?
1
Not at all
2
3
4
5
Perfectly
130
Q

RACP 2013 P2 Q67

Question 67
Mrs Higginbottom is on natalizumab for multiple sclerosis. She reports feeling “a wee bit off, love.” She thus has an MRI brain which looks like this. What is the best test to diagnose her brain infection?

a. Cryptococcal antigen.
b. Toxoplasma culture
c. John Cunningham virus PCR
d. Herpes simplex 1 virus PCR
e. Mycoplasma culture

A
How well did you know this?
1
Not at all
2
3
4
5
Perfectly
131
Q

RACP 2013 P2 Q86

Question 86
25 year old female presents with an acute, painful, swollen right knee. Other examination findings are some pustular skin lesions. Synovial fluid analysis shows-
Cell count: >90000
Cell type: >90% Neutrophils. (There was no mention about gram stain/culture)

Which of the following is the most likely organism?
A) Staphylococcus aureus
B) Mycoplasma pneumoniae
C) Mycobacterium tuberculosis
D) Neisseria gonorrhoea
E) Streptococcus pneumonae

A
132
Q

RACP 2012 Q4

A person has 10 loose bowel motions per day. He does not have a history of
recent travel. Fecal microscopy revealed leukocytes and red cells. What is the
most likely cause?

A. Campylobacter jejuni
B. Giardia lambdia
C. Clostridium difficile
D. Enterotoxigenic E. coli
E. Rotavirus

A
133
Q

RACP 2012 Q17

In which cardiac pathology should antibiotic prophylaxis be used during a tooth
extraction?

A. MV prolapse
B. Atrial septal defect
C. Aortic stenosis
D. Prosthetic valve
E. Tricuspid regurgitation

A
134
Q

RACP 2012 Q24

The most effective way of increasing sensitivity of blood cultures is to:

A. Be careful with skin preparation
B. Use a new needle when transferring blood into blood culture bottles
C. Put at least 10 ml of blood in each culture bottle
D. Only take blood cultures when patient spikes a temperature of >38.5
degrees Celsius
E. Use culture bottles that contain antimicrobial binding resin

A
135
Q

RACP 2012 Q28

What is the pathogenesis behind E.Coli causing pyelonephritis?

A. Attachment to urothelium
B. Enterotoxin
C. Resistance to amoxicillin
D. Splitting of urea
E. Survival in acidic environment

A
136
Q

RACP 2012 Q64

A pathogenic strain of Clostridium difficile produces a binary toxin. Clostridium
difficile which produces the binary toxin is more likely to:

A. Respond only to moxifloxacin
B. Infect the elderly
C. Produce severe abdominal pain
D. Produce toxic megacolon
E. Produce iron deficiency anaemia

A
137
Q

RACP 2012b Q9

What is the best treatment for this patient?

A. Flucloxacillin
B. Metronidazole
C. Ivermectin
D. Steroids
E. Albendazole

A
138
Q

RACP 2012b Q26

A 52 year old female, has poorly controlled seropositive rheumatoid arthritis. She
presents for management of an ulcer, which despite regular dressing has not healed. What is the most likely underlying pathology for the ulcer?

A. Infection
B. Pressure
C. Vasculitis
D. Venous insufficiency
E. Neuropathic

A
139
Q

RACP 2012B Q 28

It is recommended that amoxicillin and clavulanic acid should be taken with food. What is the main reason for this?

A. Decrease breakdown of amoxicillin
B. Increase absorption of amoxicillin
C. Increase absorption of clavulanic acid
D. Decrease gastrointestinal side effects
E. Decrease breakdown of clavulanic acid

A
140
Q

RACP 2012b Q31

A 56 year old executive travelled to Hanoi for work and stayed for 3 months. Four
months after his return, he developed 1 week episodes of swinging fevers and rigors.
Investigations reveal

Hb 135
WCC 11.5
Plt 155
LFTs mild mixed picture.
CXR normal
CT-scan abdomen image shows R lobe liver hypodense lesion

What should be prescribed?
A. Arthemeter and lumefantrine
B. Albendazole
C. Flucloxacillin
D. Metronidazole
E. Ivermectin

A
141
Q

RACP 2012B Q55

A male returns after a 10 day business trip from Hanoi. He presents with fevers and
rigors. He ate from several food stalls on the street while he was there. He denies engaging in unprotected sex on the trip. There is no diarrhoea. Immunochromatographic tests for malaria were negative. Blood cultures were positive for a gram negative bacillus. What is the most likely organism?

A. Dengue virus
B. Salmonella Typhi
C. Plasmodium Knowlesi
D. Entamoeba histolytica
E. Klebsiella Pneumoniae

A
142
Q

RACP 2012b Q61

Diarrhoeal illness less than 12 hours after ingestion most likely results from which food borne pathogen:
A. E coli
B. Vibrio
C. Campylobacter
D. Salmonella
E. Bacillus

A
143
Q

RACP 2012b Q69

Neuraminidase inhibitors are used in the treatment of influenza A and B to shorten
the duration of illness. When do you need the reduce the dose given to patients?

A. Age > 60
B. Renal function < 30ml/min
C. Hepatitis
D. Lymphopaenia

A
144
Q

RACP 2012b Q75

An elderly gentleman presents to hospital with symptoms and signs of a chest
infection. Urinary strep antigen is positive. He is treated for a strep pneumonia with
IV penicillin G. After 5 days of treatment he is still unwell with spiking fevers and
shortness of breath and his chest xray is as follows:

What is the next best management plan?
A. Add azithromycin
B. Change antibiotics to ceftriaxone
C. Chest physiotherapy
D. Perform CTPA
E. Arrange thorascopic drainage

A
145
Q

RACP 2012b Q80
A 46 year old has a history of throat swelling to cephalexin 20 years previously. He
has been diagnosed with endocarditis - Strep viridians sensitive to penicillin. What should you do?

A. Oral challenge to penicillin
B. Skin prick testing to penicillin
C. Commence meropenem
D. Commence vancomycin
E. Immediately desensitise to penicillin

A

Answer D (or E)

For adults with uncomplicated or complicated native valve endocarditis caused by viridans streptococci or S. bovis group with penicillin MIC 0.125 mg/L or lower who have immediate severe or delayed severe hypersensitivity to penicillins, use: vancomycin intravenously for 4 weeks

Desensitisation is appropriate only in cases of immediate hypersensitivity. Unclear from the stem if immediate or delayed hypersensitivity to penicillin.

146
Q

RACP 2012b Q95

A 45 year old doctor has had a two week history of a sore throat. He has self-
prescribe a five day course of amoxycillin but his symptoms have not resolved. He is now finding it very difficult to swallow. Examination of his throat reveals the following:

What is the likely diagnosis?
A. Epstein Barr Virus
B. Tonsillar lymphoma
C. Streptococcal pharyngitis
D. Quinsy
E. Acute tonsillitis

A
147
Q

RACP 2012b Q100

A nurse receives a needlestick injury during an operation on an HIV positive patient
who has a high viral load and is known to have been non-compliant with his
antiretroviral medication. What is the best course of action?

A. Await results of HIV genotype resistance testing prior to treatment
B. Enrol the nurse in a clinical trial of post-exposure prophylaxis
C. Treat with antiretroviral drugs that the patient has not previously been
exposed to
D. Treat with the same antiretroviral drug regime that the patient is currently on
E. Reassure the nurse that the risk of transmission from such a needlestick injury
is low

A
148
Q

RACP 2011a Q 24

The most effective way of increasing the sensitivity of blood cultures is to:

A. Be careful with skin preparation
B. Use a new needle when transferring blood into blood culture bottles
C. Put at least 10 mL of blood in each culture bottle
D. Only take blood cultures if patient spikes a temperature > 38.5
degrees Celsius
E. Use culture bottles that containing an antimicrobial binding resin

A
149
Q

RACP 2011a Q27

Which valve defect is most commonly associated with Infective Endocarditis?

A. Mitral stenosis
B. Mitral valve prolapsed
C. Aortic stenosis
D. Tricuspid regurgitation
E. Aortic regurgitation

A

Answer ? B

Mitral valve disease, such as mitral valve prolapse (usually with coexistent mitral regurgitation) and/or mitral annulus calcification, is a risk factor for IE. Two reports of IE noted mitral valve prolapse was the underlying cardiac lesion in 22 and 29 percent of cases. The risk of IE in patients with mitral valve prolapse and associated regurgitation is estimated to be five to eight times higher than that in patients with a normal mitral valve

Aortic valve disease (sclerosis, stenosis, and/or regurgitation) occurs in 12 to 30 percent of IE cases

The risk of IE appears to be low in adults with pulmonary or tricuspid regurgitation due to pulmonary hypertension in the setting of inherently normal pulmonic and tricuspid valves

Among congenital defects - IE occurred most frequently among patients with aortic stenosis and ventricular septal defects

150
Q

RACP 2011a Q30

A 35yr old man presents with an acute onset of diarrhoea, passing more than 10 stools a day. Microscopic examination shows leukocytes and erythrocytes. He has had no history of recent travel.
What is the most likely cause?

A. Campylobacter jejuni
B. Clostridium perfringens
C. Enterotoxigenic E. Coli
D. Rotavirus
E. Giardia lamblia

A
151
Q

RACP 2011a Q55

NORSA (Community-acquired, non-multiresistant oxacillin-resistant
Staphylococcus aureus) is an increasingly common cause of severe skin
abscesses and infection. In addition to incision and drainage, which anti-
microbial agent should be used?

A. Cephazolin
B. Ciprofloxacin
C. Clindamycin
D. Doxycycline
E. Erythromycin

A
152
Q

RACP 2011b Q6

36, in the ICU f or treatment of cholangitis related sepsis, received cephalosporin
and gentamicin which levels were measured 2nd daily and were within norm al ranges,
developed acute kidney injury with his creatinine doubling f rom 70 to 150 m mol/L . He has past 2.4 L of urine in the past 24 hrs. His urinalysis showed hyaline casts. T he
most likely cause of his renal impairment is:

A. Aminoglycoside toxicity
B. Glomerulonephritis
C. Hepatorenal syndrome
D. Acute tubular necrosis
E. Acute interstitial nephritis

A
153
Q

RACP 2011b Q11

Question 11
A 30 year old day care work er presenting with red eye, fever and pharyngitis. What is the most likely causative agent?

A. adenovirus
B. HSV
C. CMV
D. Leptospirosis
E. Streptococcus pyogenes

A
154
Q

RACP 2011b Q14

39F, presented with 12h of conf usion associated with he adach e and m ild
photophobia. CT brain is normal. LP: CSF protein 0.6g/l (norm al range <0. 45),cell
count 8 lymphocytes (<2 cells) and norm al glucose. T he most appropriate next step of her management is:

A. Admit and treat with I V Acyclovir
B. Admit and treat with I V Ceftriaxone
C. Admit f or observation
D. Discharge with analgesia
E. Discharge with reassurance

A
155
Q

RACP 2011b Q48

A m ale patient with a history of intravenous drug use presents with a rash, myalgias , arthralgias. He is afebrile. He is positive to hepatitis C antibody and rheumatoid
factor. He C4 complement level is reduced with normal C3 levels. His ANA is positive
at 1:160 speckled. Hi s rash is shown below:

W hat is the most likely diagnosis?
A. Cryoglobulinaemic vasculitis
B. Henoch- Schoenlein purpura
C. Rheumatoid vasculitis
D. Subacute bacterial endocarditis
E. Meningococcal disease

A
156
Q

RACP 2011b Q52

A young homosexual man presents with a painless perianal ulcer 3 weeks after
casual intercourse. Urine f or Chlamydia and gonorrhoea PCR are negative. Swab f or
HSV DNA b y PCR is negative. Syphilis serology is negative.
What is the most likely diagnosis?

A. Chancroid ulcer due to haemophilus ducreyi
B. Early syphilis
C. Genital herpes
D. Gonorrhoea
E. Lymphogranuloma venereum

A
157
Q

RACP 2011b Q70

A 40 year old lady recently returned from Southeast Asia. Presents with fever and vomiting associated with right loin pain. UA showed pyuria with WCC > 100. Culture grew Klebsiella pneumonia which is resistant to gentamicin and expresses extended spectrum beta-lactamase. What is the most appropriate initial treatment?

A. Ceftriaxone
B. Piperacillin-tazobactam
C. Meropenem
D. Linezolid
E. Norfloxacin

A
158
Q

RACP 2011b Q88

A nurse receives a needlestick injury during an operation on an HIV positive patient who has a high viral load and is known to have been non - compliant with his anti-retroviral
antiretroviral medication. What is the best course of action?

A. Await results of HIV resistance testing prior to treatment
B. Enrol the nurse in a clinical trial of post-exposure prophylaxis
C. Treat with antiretroviral drugs that the patient has not previously been exposed to
D. Treat with the same antiretroviral drug regime that the patient is currently on
E. Reassure the nurse that the risk of transmission from such a needlestick injury is low

A
159
Q

RACP 2011b Q 93

A young man visiting from South America presents with an 8 week hx of weight loss, fever and non -productive cough. You suspect tuberculosis. What test would be most sensitive for this diagnosis?

A. Bronchoscopy with bronchial washings
B. Pleural biopsy
C. Pleural effusion aspiration
D. Serum interferon gam m a release assay
E. Sputum for acid fast bacilli (AFB) stain

A
160
Q

RACP 2011b Q 97

Man post ENT surgery presents with meningism Lumbar puncture shows gram positive diplococci on gram stain , what is the most appropriate initial treatment?

A. Vancomycin and Ceftriaxone
B. Benzyl penicillin and Acyclovir
C. Ceftriaxone
D. Gentamicin and metronidazole
E. Benzyl penicillin and Ceftriaxone

A
161
Q

RACP 2010 Q4

What is the greatest risk factor for severe respiratory disease associated with the 2009 H1N1 Swine Flu ?

A. Age >70
B. BMI <20
C. Colonisation with staph aureus
D. Pregnancy
E. Previous treatment with Oseltamivir

A
162
Q

RACP 2010a Q13
A patient presents with recent onset of 10 loose bowel movements per day. Stool microscopy shows leukocytes and erythrocytes. The patient has not travelled overseas. Which of the following is the most likely pathogen?

A. Campylobacter jejuni.
B. Clostridium perfringens.
C. Enterotoxigenic E. coli.
D. Giardia lamblia.
E. Rotovirus.

A
163
Q

RACP 2010a Q43

What is the main risk factor for Clostridium difficile infection?

A. Advanced age
B. Colonoscopy
C. Hospitalisation
D. Immunosuppression
E. PPI use

A
164
Q

RACP 2010a Q64

Most common pathogen to cause prothetic valve endocarditis?

A. Coag-negative Staph
B. Enterococcus
C. HACEK
D. Staph aureus
E. Strep viridians
F. Other recall options: Gram negatives / Bartonella

A

Answer A, then D?

Prosthetic valve:
- Initial 2 months of implantation - S. aureus and coagulase-negative staphylococci followed by gram-negative bacilli and Candida species (from direct contamination during surgery, abscess common). - - Between 2-12 months - coagulase-negative staphylococci, S. aureus, and streptococci, followed by enterococci (combination of delayed nosocomial + community acquired). -
- Beyond 12 months- streptococci and S. aureus, followed by coagulase-negative staphylococci and enterococci (similar to native valves)

165
Q

RACP 2010a Q67

Which of the following vaccination is relatively contraindicated in immunocompromised patient?

A. HPV
B. Influenzae
C. Pneumococcal polysaccharide
D. Tetanus toxoid
E. VZV vaccination

A
166
Q

RACP 2010b Q1

57 yr old gentleman presenting with drowsiness, confusion and word finding difficulties. LP reveals
mildly elevated protein, WCC 5, CT Brain NAD. His MRI brain (Photo shown).

Most important management:
A. Acyclovir
B. Steroids
C. Ceftriaxone
D. Thrombolysis
E. Fluconazole

A
167
Q

RACP 2010b Q4

A 24yo male has abdominal pain 2 weeks after travelling through SE Asia. Blood film shows eosinophilia. (Blood film shown plus description). What is the most likely organism:

A. P. falciparum
B. PCP
C. strongyloidiasis
D. Salmonella typhi
E. E.coli

A
168
Q

RACP 2010b Q8

A 60 year old male has a previous rash whist talking flucoxacillin. He presents with aortic valve endocarditis with Staphylococcus Aureus sensitive to flucoxacillin. He is treated with IV Cephazolin 2g every 8 hours for the past week. He develops pulmonary oedema and a new early diastolic murmur in the aortic area.
What is the best management ?

A. Add Gentamycin to Cephazolin
B. Start flucoxacillin after rapid desensitisation
C. Change to Vancomycin and Rifampin
D. Transfer to ICU for Intra-aortic balloon pump insertion
E. Urgent aortic valve repair.

A

Answer E

Indications for surgery (valve replacement or valve repair) in IE include:
- Prosthetic valve endocarditis
- Valve dysfunction leading to heart failure
- Uncontrolled infection: e.g., enlarging vegetation, persistent bacteraemia (> 7 days despite appropriate antibiotic treatment)
- Perivalvular extension or complications: e.g., abscess, pseudoaneurysm, fistula, heart block
- Fungal endocarditis
- High embolic risk: e.g., mobile vegetation > 10 mm, recurrent embolism

169
Q

RACP 2010b Q 21

A 92 yo female from high level care nursing home presents with fever, tachycardia and SOB. CXR shows RLL consolidation. What is the most likely causative organism?

A. Haemphilus Influenzae
B. Legionella
C. Klebsiella
D. Pseudamonas
E. Streptococcus Pneumoniae

A
170
Q

RACP 2010b Q23

A backpacker returned from Thailand two months ago. He is still has crampy abdominal pain, loose stools and urgency. Which of the following is the most likely cause of his symptoms?

A. Campylobacter
B. E. coli
C. Lactose intolerance
D. Post-infectious irritable bowel syndrome
E. Tropical sprue

A
171
Q

RACP 2010b Q 32

A young homosexual man presents with painless perianal ulcer 3 weeks after casual intercourse.
Urine for Chlamydia and gonorrhoea PCR negative. Swab for HSV DNA by PCR negative. Syphillis
serology is negative.
What is the most likely diagnosis?

A. chancroid ulcer due to Haemophilus ducreyi
B. early syphilis
C. genital herpes
D. gonorrhoea
E. lymphogranuloma venereum

A
172
Q

RACP 2010b Q 43

What is the most common organism causing peritonitis in peritoneal dialysis patients?

A. anaerobes
B. corynebacteruim
C. enterobacteriacae
D. fungi
E. staphylococcus species

A
173
Q

RACP 2010b Q 74

A 72 yo pt with non-hodgkins lymphoma presents to emergency 10 days after receiving R-CHOP with

a temperature of 38.7, tachycardia, hypotension and neutrophils of 0.2. Which of the following anti-
biotics would be most appropriate?

A. Ciprofloxacin
B. Tazocin
C. Ceftriaxone + Roxythromycin
D. Augmentin DF
E. VAncomycin and Rifampicin
F. Metronidazole

A
174
Q

RACP 2010b Q90

HLA B5701 testing is now performed in HIV clinics. Which one of the following reactions does it screen for?

A. Efarivenz ± CNS symptoms
B. Abacavir ± Rash/Hypersensivity reaction
C. Bactrim ± Rash
D. Tenofovir ±renal impairment
E. Adefovir ± lactic acidosis

A
175
Q

RACP 2010b Q 92

A 32 year old male undergoes ethmoidal sinus surgery. 48 hours later he develops fever, decreased conscious state and neck stiffness. LP performed shows elevated WCC, raised protein and gram positive cocci presence on gram stain. Which of the following is most appropriate initial antibiotic
therapy? / What is the best next management approach?

A. IV ceftriaxone 2 gm bd
B. IV Meropenem 500 mg tds
C. IV Benzylpenicillin 1.2 gm qid and ceftriaxone 2 gm bd
D. IV vancomycin 1 gm daily and ceftriaxone 2 gm bd
E. IV ciprofloxacin and doxycycline
F. Penicillin and Gentamicin

A
176
Q

RACP 2009 Q35

Question 35

Which of the following is likely to be present in a patient with streptococcus bovis septicaemia?

A) Gastric cancer

B) Pancreatic cancer

C) Gallbladder cancer

D) Appendiceal cancer

E) Colon cancer

A
177
Q

RACP 2009 Q53

Question 53

In the immunocompetent host, what is the most common cause of recurrent viral meningitis?

a. CMV
b. Mumps
c. HSV 1 - encephalitis
d. HSV 2 - meningitis
e. Varicella zoster

A
178
Q

RACP 2009 Q62

Question 62
Which is most likely to have bacterial contamination?

A.Blood transfusion
B.Platelet transfusion
C.Cryoprecipitate
D.FFP
E.’Chocolate Ice-Cream’ at the Coogee Bay Hotel

A
179
Q

RACP 2009a Q69

Question 69

Which infection is responsible for native valve endocarditis in IV drug users?

a. Streptococcus Pyogenes
b. Staph aureus
c. E coli
d. Staph epidermidis

A

Answer B

IVDU: S.aureus most common cause, followed by Streptococci and Enterococci

180
Q

RACP 2009b Q1

A 42 year old male was recently diagnosed with HIV when he presented with 6 months of fever, night sweats, weight loss and general malaise.

CD4 count 0.03 x 109 [0.45‐1.10]

HIV load > 100,000copies/mL

Blood cultures grow Mycobacterium avium.

He is started on combination anti‐retroviral therapy and rifabutin, ethambutol and clarithromycin. His fever and constitutional symptoms resolve within 2 weeks. However, in the next 4 weeks, his fever recurs and he develops left sided neck lumps.

The most likely diagnosis is:

A. Non‐Hodgkin’s Lymphoma

B. Immune Reconstitution Inflammatory Syndrome

C. Toxoplasma gondii

D. Bartonella henselae

E. Secondary syphilis

A
181
Q

RACP 2009b Q1

A 42 year old male was recently diagnosed with HIV when he presented with 6 months of fever, night sweats, weight loss and general malaise.

CD4 count 0.03 x 109 [0.45‐1.10]

HIV load > 100,000copies/mL

Blood cultures grow Mycobacterium avium.

He is started on combination anti‐retroviral therapy and rifabutin, ethambutol and clarithromycin. His fever and constitutional symptoms resolve within 2 weeks. However, in the next 4 weeks, his fever recurs and he develops left sided neck lumps.

The most likely diagnosis is:

A. Non‐Hodgkin’s Lymphoma

B. Immune Reconstitution Inflammatory Syndrome

C. Toxoplasma gondii

D. Bartonella henselae

E. Secondary syphilis

A
182
Q

RACP 2009b Q31

QUESTION 31

A 72 year old T2DM patient with a chronic ulcer on the dorsum of foot for the last 6 months treated with daily dressings. She has not been hospitalised in 2 years. She has no allergies. In the last 48 hours, she has had increasing inflammation around ulcer, general malaise, unstable BSLs. There is bone on view at base of the ulcer.

What would be the best empirical antibiotic treatment?

A. Vancomycin + gentamicin

B. Ticarcillin/clavulanate

C. Metronidazole

D. Moxifloxacin

E. Linezolid and ciprofloxacin

A

Answer ?E

Falls under moderate severity diabetic foot infection involves structures deeper than the skin or subcutaneous tissues (eg muscle, bone, joint, tendon) or erythema that extends more than 2 cm from the wound margin. Infection is not associated with systemic inflammatory response syndrome (SIRS)

Treatment
amoxicillin+clavulanate 1+0.2 g intravenously, 8-hourly. If the bone is infected, use a dose of 1+0.2 g intravenously, 6-hourly
For patients with immediate nonsevere or delayed nonsevere hypersensitivity to penicillins, use:
cefazolin 2 g intravenously, 8-hourly
PLUS metronidazole 500 mg intravenously, 12-hourly.

For severe diabetic foot infection
piperacillin+tazobactam 4+0.5 g intravenously, 6-hourly.
For patients hypersensitive to penicillins , use:
ciprofloxacin 400 mg intravenously, 8-hourly
PLUS EITHER clindamycin 900 mg intravenously, 8-hourly
OR lincomycin 900 mg intravenously, 8-hourly.

If oral therapy is possible, consider replacing intravenous ciprofloxacin with oral ciprofloxacin. Use:
ciprofloxacin 750 mg orally, 12-hourly.

183
Q

RACP 2009b Q 58

An 87 year old man with dementia from a nursing home has a long term in‐dwelling catheter. He is found to have 10‐50 WCC/field on microscopy and Candida albicans on culture. He has no symptoms of urinary infection.

What is the most appropriate next step?

A. Do nothing

B. Change the catheter and administer a single dose of fluconazole

C. Change the catheter and administer IV amphotericin B

D. Leave the catheter in and commence fluconazole

E. Leave the catheter in and commence IV caspofungin

A
184
Q

RACP 2008a Q22
QUESTION 22

Patients infected with strains of Staphylococcus aureus which secrete the Panton-Valentine Leucocidin (PVL) toxin are most likely to present with which of the following?

A. Endocarditis.

B. Toxic shock syndrome.

C. Pyogenic skin infections.

D. Stevens-Johnson syndrome.

E. Intravenous catheter sepsis

A
185
Q

RACP 2008a Q33
QUESTION 33

Linezolid is most commonly associated with which of the following adverse reactions?

A. Interstitial nephritis.

B. Bullous skin eruptions.

C. Myelosuppression.

D. Cholestatic jaundice.

E. Myaesthenia gravis

A
186
Q

RACP 2008a Q42
QUESTION 42

A 42-year-old woman presents to the Emergency Department with a febrile illness with no localising symptoms or signs. After 48-hours of incubation the laboratory reports the growth of Staphylococcus epidermidis in the aerobic bottle of a blood culture set. What is the most likely explanation of this result?

A. Skin contamination.

B. Native valve endocarditis.

C. Intrahepatic abscess.

D. Urinary tract infection.

E. Periodontic disease.

A
187
Q

RACP 2008a Q59

QUESTION 59

Which of the following antibiotics is the commonest cause of Clostridium difficile colitis?

A. Clindamycin.

B. Metronidazole.

C. Gentamicin.

D. Amoxycillin.

E. Doxycycline.

A
188
Q

RACP 2008a Q62

Many human immunodeficiency virus (HIV) protease inhibitors are marketed in a combination that includes a small dose of ritonavir eg. Lopinavir 133mg-ritonavir 233mg. Which of the following provides the best rationale for the use of such combinations?

A. Enhancement of anti-HIV activity.

B. Improved patient compliance.

C. Improved pharmacokinetic profile.

D. Reduction of metabolic side effects.

E. Reduction in development of HIV drug resistance.

A
189
Q

RACP 2008a Q13

In preventing the recurrence of rheumatic fever in young patients with rheumatic heart disease, which is the most effective strategy?

A. Early treatment of throat infection.

B. Vaccination against common types of streptococcus.

C. Secondary prophylaxis with benzathine penicillin G.

D. Secondary prophylaxis with amoxycillin

E. Topical treatment of impetigo in family members.

A
190
Q

RACP2008b Q16

A 50-year-old male has a two month history of frequent, small volume stools associated with cramps and flatulence. This has occurred since returning from a holiday in South East Asia. The empiric treatment most likely to be successful is:

A. loperamide.

B. tinidazole.

C. albendazole.

D. bismuth subsalicylate.

E. norfloxacin.

A
191
Q

RACP 2008b Q23

QUESTION 23

A 62-year-old non-smoking woman is admitted with a 24-hour history of fever, rigors, cough and breathlessness. A left lower lobe pneumonia is diagnosed on chest X-ray and she is commenced on intravenous ampicillin and oral clarithromycin. The next day she has improved clinically and Streptococcus pneumoniae is isolated from blood cultures taken on admission. The isolate is found to have a minimal inhibitory concentration (MIC) for penicillin of 0.125 mg/L (intermediate resistance).

Which of the following is the most appropriate antibiotic for ongoing management?

A. Vancomycin.

B. Moxifloxacin.

C. Ceftriaxone.

D. Ampicillin.

E. Cephalothin.

A
192
Q

RACP 2008b Q28

A 72-year-old woman with type 2 diabetes presents with a chronic ulcer on the dorsum of her left foot. She has not been hospitalised in the past two years. The ulcer has been present for six months and
has been treated with daily dressings. In the last 48 hours the surrounding skin has become inflamed, she has developed general malaise and her glycaemic control has worsened. Bone is visible at the base of the ulcer. There is no history of antibiotic allergy. What is the most appropriate empiric
treatment pending the result of a swab?

A. Vancomycin and gentamicin.

B. Ticarcillin/Clavulanate.

C. Metronidazole.

D. Moxifloxacin.

E. Linezolid and ciprofloxacin.

A

Answer ? E

193
Q

RACP 2008b Q50

A 56-year-old man with a past history of bicuspid aortic valve develops bacteraemia with Staphylococcus aureus and echocardiography shows a 1.0 cm vegetation on the aortic valve. He has a known history of penicillin hypersensitivity – he reports a sudden onset of tongue and throat swelling
after receiving the drug when he was 20 years old.
What is the most appropriate intravenous antibiotic?

A. Ceftriaxone.

B. Meropenem.

C. Vancomycin.

D. Clindamycin.

E. Flucloxacillin.

A
194
Q

RACP 2008b Q 83

A 27-year-old medical student presents with three episodes of fevers, headache, chills and rigors and loose stools over the past week. She returned from an elective placement in Papua New Guinea six weeks previously. She took doxycycline for malaria prophylaxis. Examination reveals a fever of 40oC, but no other abnormal findings. Her chest X-ray is clear. The following test results were obtained:

Haemoglobin (Hb) 115 g/L [115 – 135 g/L]
White cell count (WCC) within normal limits
Platelet count 120 x 109/L [150 – 400 x 109/L]
Thick and thin blood films no parasites detected
Liver function tests within normal limits
In addition to blood cultures, what is the most appropriate next step?

A. Repeat thick and thin blood film.

B. Dengue polymerase chain reaction (PCR).

C. Mantoux test.

D. Oral metronidazole.

E. Oral ciprofloxacin.

A
195
Q

RACP 2008b Q 92

A 20-year-old male with known human immunodeficiency virus (HIV) infection is admitted from the community with a two day history of cough and sputum production. On examination he has a temperature of 38oC and his chest X-ray shows patchy opacification of the right middle and lower
lobes. The most likely organism responsible for his clinical features is:

A. Streptococcus pneumoniae.

B. Legionella pneumophila.

C. Mycobacterium avium-complex

D. Pneumocystis jiroveci (carinii).

E. Cytomegalovirus (CMV).

A
196
Q

RACP 2007a Q1

QUESTION 1

Which of the following is the most common precipitating illness leading to the Guillain-Barre syndrome?

A. Campylobacter jejuni gastroenteritis.

B. Herpes simplex infection.

C. Epstein-Barr virus infection.

D. Mycoplasma pneumonia.

E. Cytomegalovirus infection

A
197
Q

RACP 2007a Q23

Recurrent infections with which of the following organisms is most characteristic of a disorder of neutrophil phagocytic function (such as chronic granulomatous disease)?

A. Non-tuberculous mycobacteria.

B. Pseudomonas aeruginosa.

C. Nocardia species.

D. Pneumocystis jiroveci (carinii).

E. Staphylococcus aureus.

A
198
Q

RACP 2007a Q26

The emergence of a pandemic strain of influenza is best explained by which of the following mechanisms?

A. Antigenic drift in H and N proteins of influenza A.

B. Antigenic shift in H and N proteins of influenza A.

C. Point mutations in the M proteins of influenza A.

D. Development of neuraminidase resistance.

E. Recombination of influenza A and B haemagglutinins

A
199
Q

RACP 2007a Q32

Many HIV (human immunodeficiency virus) protease inhibitors are marketed in a combination that includes a small dose of ritonavir eg. Lopinavir 133mg-ritonavir 233mg. Which of the following provides the best rationale for the use of such combinations?

A. Enhancement of anti-HIV activity.

B. Improved patient compliance.

C. Improved pharmacokinetic profile.

D. Reduction of metabolic side effects.

E. Reduction in development of HIV drug resistance

A
200
Q

RACP 2007a Q37

In a patient with rheumatic mixed mitral valve disease, for which of the following procedures is there the strongest indication for antibiotic prophylaxis?

A. Flexible bronchoscopy.

B. Cardiac catheterisation.

C. Colonoscopy without biopsy.

D. Oesophageal dilatation.

E. Gastroscopy.

A
201
Q

RACP 2007a Q40

Patients infected with strains of Staphylococcus aureus which secrete the Panton-Valentine Leucocidin (PVL) toxin are most likely to present with which of the following?

A. Endocarditis.

B. Toxic shock syndrome.

C. Pyogenic skin infections.

D. Stevens-Johnson syndrome.

E. Intravenous catheter sepsis.

A
202
Q

RACP 2007a Q45

QUESTION 45

A patient presenting to the Emergency Department is thought to have acute intermittent porphyria. Which of the following is most suggestive of an alternative diagnosis?

A. Ileus.

B. Hypertension.

C. Fever.

D. Urinary retention.

E. Tachycardia.

A
203
Q

RACP 2007a Q63

QUESTION 63

The first manifestation of illness in a patient with new variant Creutzfeldt-Jacob Disease (nvCJD) is most likely to be which of the following?

A. Psychiatric.

B. Myoclonus.

C. Ataxia.

D. Slurred speech.

E. Epilepsy.

A
204
Q

RACP 2007a Q67

QUESTION 67

Human papilloma virus (HPV) is a well-recognised causal factor for cancer of the cervix. For which of

the following other cancers is there the strongest association with HPV?

A. Uterine.

B. Anal.

C. Rectal.

D. Oral.

E. Vulval.

A
205
Q

RACP 2007b Q4

QUESTION 4

Which of the following factors is the strongest contraindication to the use of interferon in the treatment of viral hepatitis:

A. disease acquisition during childhood.

B. Child-Pugh category C status.

C. serum alanine transaminase levels <2 times normal.

D. hepatitis e antigen negative infection.

E. immunocompromised status.

A
206
Q

RACP 2007b Q7

QUESTION 7

A 38-year-old man, born in Thailand and living in Australia for the past two years, presents to the emergency department with his first generalized seizure. A head CT scan is performed as shown below and shows multiple calcified lesions. Management should focus on which of the following?

A. Seizure prevention.

B. Parasite eradication.

C. Transmission reduction.

D. Surgical removal.

E. Bowel decontamination.

A
207
Q

RACP 2007bQ 20

QUESTION 20

A 60-year-old man has been identified as HIV (human immunodeficiency virus) positive for at least 15 years. On antiretroviral therapy, his viral load is undetectable and his CD4 count is 220. He also has vascular risk factors. His partner describes memory impairment of six months duration and a change in personality. On mental state examination, he is irritable but is attentive and orientated. He has an expressive dysphasia and some impairment of short-term memory. A CT scan of the brain demonstrates mild cerebral atrophy but no evidence of opportunistic infection or cerebral malignancy. Which of the following features in the history, examination and investigations is most suggestive of a diagnosis other than HIV dementia as the cause of his cognitive impairment?

A. Short-term memory impairment.

B. Dysphasia.

C. Personality change.

D. CD4 count.

E. Undetectable viral load

A
208
Q

RACP 2007b Q24

QUESTION 24

A 21-year-old man with a history of injecting drug use is admitted to the Intensive Care Unit with a severe bilateral pneumonia. He has a recent history of recurrent skin boils which have been difficult totreat. He has received a number of courses of oral antibiotics for these lesions with little effect. He is intubated and gram stain of the endotracheal aspirate shows Gram positive cocci. Sputum cultures show a heavy growth of Staphylococcus aureus with sensitivities to follow. He is treated with intravenous ceftriaxone and a macrolide.

What is the most appropriate additional treatment?

A. Flucloxacillin.

B. Rifampicin.

C. Meropenem.

D. Vancomycin.

E. Gentamicin.

A
209
Q

RACP 2007B Q35

QUESTION 35

A 42-year-old woman with human immunodeficiency virus (HIV) infection presents to the emergency department with a six week history of general malaise, fever, night sweats and loss of 5kg weight. She
was born in Papua New Guinea but came to Australia five years ago. She takes zidovudine, lamivudine and nevirapine. Her viral load is undetectable and CD4 count is 0.41 x 109/L [0.45-1.4 x 109/L]. Examination is unremarkable.

Her Chest X-ray shows patchy consolidation in the left upper zone as shown below.

What is the most likely cause of the radiological changes?

A. Pneumocystis pneumonia (PCP).

B. Mycobacterium avium complex (MAC).

C. Mycobacterium tuberculosis (MTB).

D. Burkholderia pseudomallei.

E. Cytomegalovirus (CMV).

A
210
Q

RACP 2007b Q55

QUESTION 55

A 45-year-old human immunodeficiency virus-1 (HIV-1)-infected male who has been followed without anti-retroviral therapy for 12 months is referred for assessment and further management. He has no HIV-related symptoms or signs. Baseline evaluation and counselling is performed. Which of the following findings provides the strongest indication to initiate anti-retroviral therapy
immediately?

A. CD4 cell count 300 per cubic millimeter.

B. Plasma HIV-1 viral load 100,000 copies/mL.

C. Decline of CD4 cell count from 350 to 300 per cubic millimetre over 12 months.

D. Evidence of drug-resistant HIV variants in HIV genotyping.

E. Active hepatitis C infection on serologic testing

A
211
Q

RACP 2007b Q58

QUESTION 58

A 45-year-old woman receiving total parenteral nutrition for short bowel syndrome through a Hickman’s catheter presents with fever, sweats and rigors. Three sets of blood cultures grow Candida albicans.

Which of the following is the most appropriate management?

A. Remove the catheter, no antibiotics required.

B. Remove the catheter, commence flucytosine.

C. Leave catheter in situ, commence voriconazole.

D. Leave catheter in situ, commence amphotericin B.

E. Remove the catheter, commence amphotericin B.

A
212
Q

RACP 2007b Q61

QUESTION 61

A 30-year-old previously fit childcare worker presents with a four-week history of a non-productive cough. Clinical examination is normal. The most likely diagnosis is:

A. Gastro-oesophageal reflux disease.

B. Hypersensitivity pneumonitis.

C. Post nasal drip syndrome.

D. Pertussis infection.

E. Asthma.

A
213
Q

RACP 2007b Q72

QUESTION 72

A 62-year-old non-smoking woman is admitted with a 24-hour history of fever, rigors, cough and breathlessness. A left lower lobe pneumonia is diagnosed on Chest X-ray and she is commenced on intravenous ampicillin and oral clarithromycin. The next day she has improved clinically and Streptococcus pneumoniae is isolated from blood cultures taken on admission. The isolate is found to have a minimal inhibitory concentration (MIC) for penicillin of 0.125mg/L (intermediate resistance).

What is the most appropriate antibiotic for ongoing managemnet?

A. Vancomycin.

B. Moxifloxacin.

C. Ceftriaxone.

D. Ampicillin.

E. Cephalothin.

A
214
Q

RACP 2007b Q65

67-year old diabetic woman has end-stage renal failure managed by continuous ambulatory peritoneal dialysis. She presents with abdominal pain, low grade fever and cloudy dialysate. On examination she has mild generalised abdominal tenderness and guarding but does not appear particularly unwell. She is admitted to the ward and started on intra-peritoneal cephalexin. The subsequent day, her dialysate culture grows enterococci, E coli and Klebsiella species.

The most appropriate management step is:

A. exploratory laparotomy.

B. add intraperitoneal Ampicillin.

C. removal of the Tenckhoff catheter.

D. intraperitoneal gentamicin.

E. change to intravenous antibiotics.

A
215
Q

RACP 2007b Q77

A 90-year-old female has osteoarthritis and early dementia. She is living at home with her daughter who is her full time carer. Screening tests on her 90th birthday include an mid stream urine (MSU), which showed bacteriuria (mixed growth) and pyuria.

Which of the following strategies is most appropriate?

A. Single dose of amoxycillin.

B. Five day course of trimethoprim.

C. Topical estrogen cream.

D. No intervention.

E. Repeat MSU in three months.

A
216
Q

RACP 2007b Q79

A 72-year-old man admitted to hospital with a liver abscess was initially treated with intravenous ceftriaxone alone. Cultures from a percutaneous aspirate specimen subsequently grow Streptococcus
milleri. On day seven he developed profuse, watery diarrhoea (eight bowel actions per day) associated with fever and abdominal pain. A stool specimen has been taken. In addition to changing to a narrow
spectrum antibiotic, what is the most appropriate treatment while waiting for the result?

A. Intravenous clindamycin.

B. Oral vancomycin.

C. Oral metronidazole.

D. Oral ciprofloxacin.

E. Intravenous vancomycin.

A
217
Q

RACP 2007b Q 82

A 56-year-old farmer with a history of chronic mid and lower back pain is admitted with a two week history of fevers, rigors, general malaise and excruciating mid-thoracic back pain. On examination he has a temperature of 38.5 degrees Celsius and tenderness to palpation over the 6th and 7th thoracic
vertebrae. The rest of the examination is unremarkable. Full blood count shows a white cell count of
13.0 x 10^9/L [4.0–11.0] with a neutrophilia [10.2 x 10^9/L]. C reactive protein is 190 u/L [<5]) and ESR 69 mm/hr [<20]. Blood cultures are pending.

Which of the following investigations is most likely to establish the diagnosis?

A. Radionuclide bone scan.

B. MRI of spine.

C. CT scan of spine.

D. Plain X-rays of spine.

E. Labelled white cell scan.

A

Answer B

Stem suspiscious for vertebral OM - preferred imaging of choice is MRI spine.

218
Q

RACP 2007b Q93

A 26-year-old man recently returned from an overseas holiday, presents with a two day history of joint pain. On examination his temperature is 38.1oC, he has two small pustular lesions on his left hand,
evidence of tenosynovitis of the left 4th flexor tendon and an effusions in the right knee.

Which diagnosis best explains this clinical pattern?

A. Reactive arthritis.

B. Parvovirus infection.

C. Gonococcal arthritis.

D. Staphylococcal arthritis.

E. Sub acute bacterial endocarditis.

A
219
Q

RACP 2006a Q41

An 18-year-old male develops a rash and becomes critically ill.

The rash is demonstrated above. The most likely finding on blood cultures would be:

A. gram negative rods.

B. gram positive rods.

C. gram negative diplococci.

D. gram positive diplococci.

E. gram positive cocci.

A
220
Q

RACP 2006a Q41

QUESTION 46

The emergence of a pandemic strain of influenza is best explained by which of the following mechanisms?

A. Antigenic drift in H and N proteins of influenza A.

B. Antigenic shift in H and N proteins of influenza A.

C. Point mutations in the M proteins of influenza A.

D. Development of neuraminidase resistance.

E. Recombination of influenza A and B haemagglutinins

A
221
Q

RACP 2006 Q65

Following kidney transplantation, which of the following is the most likely manifestation of BK virus infection?

A. Aplastic anaemia.

B. Pulmonary infiltrates.

C. Nephropathy.

D. Progressive multifocal leucoencephalopathy.

E. Systemic vasculopathy.

A
222
Q

RACP 2006a Q68

Resistance of gram-positive organisms to penicillin is most commonly mediated through which of the following mechanisms?

A. Antibiotic efflux.

B. Beta-lactamase production.

C. Alteration of the ribosomal target.

D. Modification of penicillin binding proteins.

E. Decreased outer membrane permeability.

A
223
Q

RACP 2006b Q40

QUESTION 40

A 26-year-old man presents with a swollen, painful left ankle and right wrist ten days after being treated with doxycycline for chlamydial urethritis. Which of the following is the most appropriate for the initial treatment of his joint symptoms?

A. Azithromycin.

B. Prednisolone.

C. Sulfasalazine.

D. Indomethacin.

E. Hydroxychloroquine.

A
224
Q

RACP 2006b Q56

QUESTION 56

A 62-year-old man is admitted to hospital with fevers, malaise and myalgias six weeks after a laparoscopic cholecystectomy. On examination he has a temperature of 39°C, splinter haemorrhages and a loud pansystolic murmur. He has a past history of mitral valve prolapse which was diagnosed by echocardiography. Enterococcus faecalis has been identified in three sets of blood cultures. The isolate is highly sensitive to penicillin.

He has no known allergies.

The most appropriate therapy is:

A. ceftriaxone.

B. vancomycin alone.

C. ampicillin alone.

D. ampicillin and gentamicin.

E. cephalothin and gentamicin.

A
225
Q

RACP 2006a Q63

QUESTION 63

A 36-year-old male with a history of recent injecting drug use presents with a week of fevers and sweats and excruciating lower thoracic back pain. Blood cultures grow Staphylococcus aureus. What would be the most useful next investigation to establish a diagnosis?

A. CT (computed tomography) scan of thoracolumbar spine.

B. MRI (magnetic resonance imaging) scan of thoracolumbar spine.

C. Lumbar puncture.

D. Radionuclide bone scan.

E. Labelled white cell scan.

A

Answer B

Case stem suggestive of haematogenous source of OM to the vertebrae (most common site of haematogenous OM, IVDU is risk factor)

Investigations:
- MRI of spine - looking for any complications e.g. epidural abscesses
- Bone biopsy - to identify causative organism

CT with contrast and nuclear imaging done in people whom MRI is contraindicated.

226
Q

RACP 2006b Q68

QUESTION 68

A physician becomes ill with nausea and vomiting four hours after attending a pharmaceutical company sponsored dinner.

Which of the following is the most likely cause of this?

A. Clostridium difficile.

B. Staphylococcus aureus.

C. Escherichia coli.

D. Vibrio parahemolyticus.

E. Campylobacter jejuni.

A
227
Q

RACP 2006b Q78

QUESTION 78

A 35-year-old man is admitted to hospital with a red and tender left thigh, three days after sustaining a superficial abrasion to the area while surfing. He gives a history of fevers and chills and says that the pain in the leg is now increasing. On examination he appears unwell, is hypotensive, tachycardic and febrile to 40°C. The left thigh is erythematous, te nder to deep palpation with a ‘boggy’ feel over the muscles. He is observed in the emergency department and the redness over the thigh progresses 2 cm in a two hour period. A computed tomography (CT) scan of the area shows inflammation of the muscles and loss of fascial planes, but no abscess formation. Intravenous penicillin and clindamycin are started.

Which of the following additional interventions is most likely to result in survival and limb preservation?

A. Surgical exploration and debridement.

B. Intravenous hydrocortisone.

C. Intravenous immunoglobulin.

D. Hyperbaric oxygen therapy.

E. Intravenous activated protein C

A
228
Q

RACP 2005a Q20
QUESTION 20

Which of the following best describes the antiviral effect of zanamivir on the influenza virus?

A. Inhibition of M2 ion channel activity.

B. Inhibition of viral neuraminidase.

C. Competitive blocking of surface haemagglutinins.

D. Inhibition of reverse transcriptase.

E. Inhibition of DNA polymerase.

A
229
Q

RACP 2005a Q45

QUESTION 45

Which of the following is the most important reason for not recommending gentamicin for the treatment of coliform central nervous system (CNS) infections?

A. It is not active in an acidic environment.

B. It is not active in a low oxygen tension environment.

C. It has poor CNS penetration.

D. It may precipitate seizures.

E. Ototoxicity risk is accentuated.

A
230
Q

RACP 2005a Q61

QUESTION 61

In the treatment of hepatitis C infection with interferon and ribavirin, which one of the following is the best predictor of sustained virological response?

A. Serum bilirubin.

B. Viral genotype.

C. Gender.

D. Viral load.

E. Portal fibrosis.

A
231
Q

RACP 2005a Q69

QUESTION 69

The most common cause of diarrhoea in an adult traveller to a developing country is:

A. Shigella species.

B. Salmonella species.

C. Campylobacter species.

D. enterotoxigenic Escherichia coli.

E. Giardia lamblia.

A
232
Q

RACP 2005b Q5

QUESTION 5

Which of the following is the strongest contraindication to the administration of influenza vaccine?

A. Egg anaphylaxis.

B. Previous local reaction to influenza immunization.

C. Recent pneumococcal immunization.

D. Pregnancy.

E. Immunodeficiency.

A
233
Q

RACP 2005b Q8

QUESTION 8

A 42-year-old man in the intensive care unit following a motor vehicle accident develops Staphylococcus aureus bacteraemia. He is commenced on vancomycin. Following the first infusion he develops a rash involving the face and upper trunk. His blood pressure falls to 100/70 mmHg.

What is the most appropriate next step?

A. Stop vancomycin, commence linezolid.

B. Stop vancomycin, commence dalfopristin/quinupristin.

C. Commence rapid desensitisation for vancomycin.

D. Continue vancomycin but slow the rate of infusion.

E. Continue vancomycin but premedicate with hydrocortisone.

A
234
Q

RACP 2005 b Q26

A 56-year-old forestry worker develops a widespread rash involving the trunk and lower limbs two weeks following a tick bite. He is generally unwell with myalgias, fevers, headache and anorexia. The rash is papular, non-blanching and not itchy, and is shown above.

What is the most appropriate management?

A. Treat expectantly.

B. Doxycycline.

C. Ciprofloxacin.

D. Ceftriaxone.

E. Phenoxymethylpenicillin (penicillin V).

A
235
Q

RACP 2005b Q 31
QUESTION 31

A 67-year-old man has recently commenced haemodialysis for renal failure as a result of chronic glomerulonephritis. He receives immunisation with three separate doses of hepatitis B vaccine according to the recommended protocol. Six weeks after the third dose, his anti-hepatitis B surface antigen antibody level is <10 mU/mL.

Which one of the following is the most appropriate course of action?

A. Reimmunise using Freund’s adjuvant.

B. Repeat anti-hepatitis B testing in 3 months.

C. No further hepatitis B immunization.

D. Give hepatitis B immunoglobulin.

E. Immunise with up to 3 further doses of hepatitis B vaccine.

A
236
Q

RACP 2005b Q35

A 35-year-old male has the following hepatitis B virus (HBV) profile with normal liver biochemistry:

He develops lymphoma requiring chemotherapy (CHOP: cyclophosphamide, doxorubicin, vincristine and prednisone).

Hep B surface Antigen +ve
Hep B e antigen -ve
Hep B DNA -ve

With regard to his HBV status, which of the following is the most appropriate strategy?

A. Monitor liver enzymes during and after treatment.

B. Prophylactic anti-viral therapy.

C. Monitor e-antigen status during therapy.

D. No specific management necessary.

E. Modify chemotherapy regimen.

A
237
Q

RACP 2005b Q41

A 54-year-old woman presents with a six-month history of intensely itchy papules and vesicles on elbows, knees and buttocks as shown above. She is systemically well other than mild diarrhoea. Blood count is reported as normal. Serum ferritin is 5 μg/L [20-380] and red blood cell folate is 180 nmol/L [600-1600].

Which of the following investigations is most likely to confirm the diagnosis of the rash shown above?

A. IgA endomysial antibodies.

B. Direct immunofluorescence of perilesional skin.

C. Light microscopy of unexcoriated vesicle.

D. Jejunal biopsy.

E. Response to gluten-free diet.

A
238
Q

RACP 2005b Q 58

QUESTION 58

A 28-year-old woman becomes unwell the day after returning from a month-long holiday in Thailand. She complains of a fever with severe headache, myalgia and sore eyes. On the fourth day her temperature settles but two days later recurs and she then develops a generalised rash.

On examination she is pyrexial with a temperature of 37.9oC. Pulse is 80/minute, and blood pressure 120/80 mmHg. There is a diffuse macular rash over the chest and abdomen. The rest of the examination is normal.

Blood tests show a normal haemoglobin, normal leukocyte count, and normal platelets. Blood film examinations for malaria parasites are negative on three occasions.

Which one of the following is the most likely diagnosis?

A. Scrub typhus.

B. Typhoid.

C. Acute schistosomiasis.

D. Dengue.

E. Leptospirosis.

A
239
Q

RACP 2005b Q87

QUESTION 87

A 22-year-old man presents with ten days of fever that started three weeks after returning from visiting family in Bangladesh. He complains of headache, abdominal pains, anorexia, a dry cough and generalised muscular aches and pains.

On examination he looks ill. Temperature is 40.5oC, pulse 92/minute and blood pressure 120/65 mm Hg. On auscultation he has a clear chest. There is generalised abdominal tenderness without rigidity or guarding.

Blood tests show:

haemoglobin 140 g/L [130-180]

white cell count 3.5 x 109/L [4.0-11.0]

Blood films are repeatedly negative for malaria. Renal function, electrolytes, liver enzymes and chest X-ray are normal.

What is the most likely diagnosis?

A. Acute schistosomiasis.

B. Amoebiasis.

C. Leptospirosis.

D. Typhoid.

E. Dengue fever.

A
240
Q

RACP 2004a Q1

QUESTION 4

A hospitalised patient with a rash on her face and a lace-like rash on her extremities has a confirmed diagnosis of erythema infectiosum (parvovirus B19).

Which of the following infection controls would be the most appropriate?

A. Standard.

B. Contact.

C. Respiratory.

D. Single room.

E. Negative pressure room.

A
241
Q

RACP 2004a Q21

QUESTION 21

Which one of the following is least likely to be associated with a reduced risk of mother-to-child transmission of human immunodeficiency virus (HIV)?

A. Maternal plasma HIV RNA concentration (viral load) <1000 copies/mL.

B. Maternal CD4+ T cell count >500 cells/mm3.

C. Maternal antiretroviral therapy.

D. Avoidance of breastfeeding.

E. Elective Caesarean delivery.

A
242
Q

RACP 2004a Q27

QUESTION 27

Which of the following antibiotics would be the most appropriate oral medication for the outpatient treatment of a wound infection caused by Escherichia coli and Pseudomonas aeruginosa?

A. Moxifloxacin.

B. Gatifloxacin.

C. Ciprofloxacin.

D. Linezolid.

E. Amoxycillin/clavulanate.

A
243
Q

RACP 2004a Q45

QUESTION 45

A 37-year-old man presents to the emergency department with symptoms of meningitis. Gram stain of the cerebrospinal fluid reveals the presence of gram-negative diplococci.

His 12-week pregnant partner should receive which one of the following as prophylaxis?

A. Ciprofloxacin.

B. Ceftriaxone.

C. Penicillin.

D. Meningococcal vaccine.

E. Erythromycin.

A
244
Q

RACP 2004b Q66

QUESTION 66

The most common cause of diarrhoea in an adult traveller to a developing country is:

A. Shigella species.

B. Salmonella species.

C. Campylobacter species.

D. enterotoxigenic Escherichia coli.

E. Giardia lamblia.

A
245
Q

RACP 2004b Q4

QUESTION 4

A 35-year-old married indigenous woman presents with a four-day history of fever, mild headache and severe polyarthritis involving the wrists, elbows and knees. Joint pain has been only slightly relieved by naproxen 250 mg twice daily and paracetamol 1-2 g daily. Prior to the onset of arthritis, she had a sore throat for several days but denies rash, vaginal discharge or history of sexually transmitted diseases.

On examination, she appears unwell with a temperature of 39°C, an erythematous throat with tonsillar enlargement and a few small tender cervical lymph nodes. There is tenderness and soft tissue swelling of the wrists and knees. Cardiovascular examination is normal.

Which of the following investigations is most likely to identify the diagnosis?

A. Blood cultures for Staphylococcus aureus.

B. A high vaginal swab for Neisseria gonorrhoeae.

C. A throat swab for Streptococcus species.

D. A stool culture for Yersinia enterocolitica.

E. A synovial fluid culture for Neisseria meningitidis.

A
246
Q

RACP 2004b Q11

QUESTION 11

Which of the following is the most likely organism causing the presentation shown in the photographs below?

A. Neisseria meningitidis.

B. Staphylococcus aureus.

C. Mycoplasma pneumoniae.

D. Listeria monocytogenes.

E. Coxsackievirus.

A
247
Q

RACP 2004b Q23

QUESTION 23

A 32-year-old man has a splenectomy following a motorcycle accident. Which of the following organisms is most likely to cause overwhelming post-splenectomy infection in this man?

A. Streptococcus pneumoniae.

B. Neisseria meningitidis.

C. Staphylococcus aureus.

D. Escherichia coli.

E. Haemophilus influenzae.

A
248
Q

RACP 2004b Q26

QUESTION 26

Which of the following antibiotics most commonly leads to oesophageal ulceration?

A. Amoxycillin.

B. Ciprofloxacin.

C. Clindamycin.

D. Doxycycline.

E. Erythromycin.

A
249
Q

RACP 2004b Q27

A 17-year-old adolescent male is referred for investigation of recurrent pyogenic skin lesions. He gives an 18-month history of recurrent boils and cutaneous abscesses. There is no family history of similar problems and the patient is otherwise in excellent health. Examination reveals evidence of multiple healed lesions of the face and neck, axillae, groin and anterior chest wall. He has received repeated courses of both oral and intravenous antibiotics, and on a number of occasions the lesions have required surgical drainage. Culture of the abscess contents on one of these occasions yielded a heavy growth of Staphylococcus aureus.

Which of the following is the most appropriate initial investigation?

A. Serum complement levels.

B. Culture of a nasal swab.

C. Serum immunoglobulin levels.

D. Neutrophil function studies.

E. Lymphocyte subsets.

A
250
Q

RACP 2004b Q36

A 35-year-old man with a history of severe penicillin allergy presents with fever and a widespread rash, part of which is shown in the photograph below.

Which of the following is the most appropriate immediate treatment?

A. Penicillin.

B. Ceftriaxone.

C. Erythromycin.

D. Ciprofloxacin.

E. Vancomycin.

A
251
Q

RACP 2004b Q53

QUESTION 53

Angiostrongylus cantonensis, the rat lung worm, is acquired by eating the intermediate snail or slug host or contaminated vegetables.

Which of the following clinical syndromes is most characteristic of this parasite?

A. Haematuria.

B. Rectal bleeding.

C. Ascending cholangitis.

D. Eosinophilic meningitis.

E. Visceral larva migrans.

A
252
Q

RACP 2004b Q 54

A 45-year-old woman presents with a three-year history of recurrent rash, characterised as palpable purpura over both legs. Some of these lesions have ulcerated.

Laboratory results include:

Which of the following is the most appropriate management?

A. Plasmapheresis and corticosteroids.

B. Corticosteroids and interferon α.

C. Interferon α and ribavirin.

D. Cyclophosphamide and corticosteroids.

E. Cyclophosphamide and ribavirin.

A
253
Q

RACP 2004b Q57

QUESTION 57

A 47-year-old man on long-term haemodialysis presents with a fever (39.5°C) and chills. He has had multiple thrombosed fistulas and a thrombosed right axillary vein. He had a left subclavian catheter inserted under ultrasound guidance three days ago.

On examination he looks well, with a blood pressure of 170/80 mmHg and an exit site of the vascular access catheter with the appearance shown below.

Which of the following management strategies would be most appropriate in his case?

A. Intravenous vancomycin and observation.

B. Immediate haemodialysis.

C. Intravenous cephalexin and observation.

D. Intravenous vancomycin and catheter removal.

E. Replace vascular catheter with a re-wire technique.

A
254
Q

RACP 2004b Q67

A 26-year-old man presents with a two-day history of fevers, rigors, headache, malaise, nausea and vomiting, dry cough, mild arthralgia and backache. He reports no shortness of breath, diarrhoea, neck stiffness or photophobia. He reports that he returned from a diving trip to the Solomon Islands seven days ago. He had taken doxycycline regularly as malarial prophylaxis.

On day 3 after the onset of the illness he develops a rash over his trunk (shown below) and face. A petechial rash was noted on his lower limbs

Which of the following is the most likely diagnosis?

A. Malaria.

B. Dengue fever.

C. Typhoid fever.

D. Q fever.

E. Measles.

A
255
Q

RACP 2004b Q76

QUESTION 76

In human immunodeficiency virus (HIV)-infected patients receiving combination antiretroviral therapy, including nucleoside reverse transcriptase inhibitors, which of the following drug side-effects is least likely to be due to mitochondrial toxicity?

A. Lactic acidosis.

B. Pancreatitis.

C. Stevens-Johnson syndrome.

D. Myopathy.

E. Peripheral neuropathy.

A
256
Q

RACP 2004b Q78

QUESTION 78

To prevent adverse drug effects, which of the following antimicrobials requires the greatest dose reduction in severe renal impairment?

A. Ticarcillin.

B. Aciclovir.

C. Cefaclor.

D. Norfloxacin.

E. Meropenem

A
257
Q

RACP 2004b Q83

QUESTION 83

Which of the following antibiotics would be the most appropriate choice for the treatment of a vancomycin-resistant Enterococcus faecalis bloodstream infection?

A. Cefoxitin.

B. Linezolid.

C. Amikacin.

D. Tobramycin.

E. Rifampicin.

A
258
Q

RACP 2004b Q99

QUESTION 99

A 30-year-old medical officer sustains a needle-stick injury while treating a trauma patient in the emergency department. She had undergone a three-dose course of hepatitis B vaccine 10 years earlier and one year following this course was documented to be hepatitis B surface antibody (HbsAb)-positive. However, on repeat testing (done as a result of this incident), the medical officer is now found to be HbsAb-negative. The source patient in the needle-stick injury is known to be an intravenous drug user, and is shown to be hepatitis B surface antigen (HbsAg)-positive on baseline serology. Human immunodeficiency virus (HIV) and hepatitis C serology are negative.

In addition to counselling, which of the following is the most appropriate immediate action following this needle-stick injury?

A. Booster hepatitis B vaccination.

B. Hepatitis B immunoglobulin.

C. Hepatitis B vaccine and hepatitis B immunoglobulin.

D. No action required.

E. Post exposure prophylaxis for HIV.

A
259
Q

RACP 2003a Q19

QUESTION 19

In patients with untreated post-transfusion hepatitis C, the lifetime risk of progression to cirrhosis best approximates:

A. 1%.

B. 5%.

C. 20%.

D. 50%.

E. 80%.

A
260
Q

RACP 2003a Q26

QUESTION 26

A 37-year-old asymptomatic man, with known human immunodeficiency virus (HIV) infection for nine years and no previous drug treatment, elects to commence anti-retroviral therapy due to a decline in his CD4 positive (CD4+) T cell count to 250 cells/µL.

On review of his CD4+ T cell counts measured three-monthly over the past six years, it emerges that a decline of 40 cells/µL/year was noted until 18 months previously when the rate of decline accelerated to 150 cells/µL/year.

Which one of the following is the most likely cause of the change in rate of decline of CD4+ T cell count?

A. A subclinical opportunistic infection.

B. Loss of thymus function.

C. Change of viral co-receptor usage.

D. Disruption of lymph node architecture.

E. Reduced interleukin 2 (IL-2) production.

A
261
Q

RACP 2003a Q34

QUESTION 34

A 39-year-old woman presents with signs and symptoms of pyelonephritis. Despite being treated for five days with oral cephalexin (500 mg four times/day), a midstream urine culture is positive for enterococci.

The most likely explanation for the lack of response to the prescribed antibiotic is:

A. lack of tissue penetration.

B. poor compliance.

C. antibiotic resistance.

D. oral rather than intravenous therapy.

E. inadequate duration of therapy.

A
262
Q

RACP 2003a Q49

QUESTION 49

A 72-year-old woman presents with severe diarrhoea. She commenced amoxycillin for an upper respiratory tract infection five days ago.

Which one of the following tests would best confirm a diagnosis of Clostridium difficile-induced pseudomembranous colitis?

A. C. difficile culture.

B. C. difficile cytotoxin assay.

C. C. difficile polymerase chain reaction (PCR).

D. C. difficile immunoglobulin A (IgA).

E. Faecal microscopy.

A
263
Q

RACP 2003a Q60

A 37-year-old man presents to the emergency department with symptoms of meningitis. Gram stain of the cerebrospinal fluid reveals the presence of gram-negative diplococci.

His 12-week pregnant partner should receive which one of the following as prophylaxis?

A. Ciprofloxacin.

B. Ceftriaxone.

C. Penicillin.

D. Meningococcal vaccine.

E. Erythromycin

A
264
Q

RACP 2003a Q66

QUESTION 66

The investigation most useful in the management of cytomegalovirus (CMV) disease in the

immunocompromised host is:

A. CMV blood culture.

B. direct CMV antigen detection in peripheral blood mononuclear cells (PBMCs).

C. CMV immunoglobulin M (IgM).

D. plasma CMV DNA concentration (viral load).

E. early antigen detection in supernatant after culture of PBMCs for 48 hours.

A
265
Q

RACP 2003b Q12

QUESTION 12

Which one of the following drugs is least effective in the treatment of Legionnaire’s disease?

A. Rifampicin.

B. Gentamicin.

C. Erythromycin.

D. Ciprofloxacin.

E. Azithromycin.

A
266
Q

RACP 2003b Q15

QUESTION 15

A 19-year-old man, who is sexually active and admits to occasional recreational drug use, presents with a three-day history of severe pain over the left sacroiliac area. Examination reveals a temperature of 38°C, pulse rate of 80/minute and blood pressure of 120/80 mmHg. The only other findings are ridged nails and pain on movement of the spine or when pressure is applied to the sacroiliac joints.

Investigations show:

X-rays of the spine and pelvis are normal. The bone scan is shown below.

The most likely diagnosis is:

A. reactive arthritis following Chlamydia trachomatis infection.

B. psoriatic arthritis.

C. gonococcal arthritis.

D. septic sacroiliitis.

E. undifferentiated spondylarthropathy.

A
267
Q

RACP 2003b Q27

QUESTION 27

A 25-year-old man presents six weeks after a trip to central Africa with fever, chills and rigors. Stool examination is positive for Entamoeba histolytica. An ultrasound reveals a 7.5 cm liver abscess.

The most appropriate next step in this patient’s management is:

A. diagnostic aspirate of the liver abscess.

B. percutaneous drainage of the liver abscess.

C. metronidazole therapy.

D. praziquantel therapy.

E. quinine therapy.

A
268
Q

RACP 2003b Q37

QUESTION 37

A hospitalised patient is seen with a stage III pressure ulcer (full thickness skin loss and necrosis of subcutaneous tissue extending down to, but not through, underlying fascia) on the sacrum that measures 10 x 10 cm. There is extensive eschar adhering to the centre and to one side of the wound and a large amount of purulent drainage is expressible from under the eschar. There is no surrounding skin erythema. The patient is afebrile.

Which one of the following approaches would best control local infection?

A. Debridement with saline irrigation and dry gauze dressing.

B. Intravenous antibiotic therapy.

C. Topical antibiotic therapy.

D. Topical antiseptics.

E. Occlusive calcium alginate dressing.

A
269
Q

RACP 2003b Q54

QUESTION 54

A 54-year-old man with methicillin-resistant Staphylococcus aureus (MRSA) sternal osteomyelitis is receiving ongoing treatment with vancomycin. Shortly after administration of his vancomycin dose, he develops diffuse erythema and hypotension.

The most likely cause is:

A. hypersensitivity to vancomycin.

B. rapid intravenous infusion of vancomycin.

C. endotoxin release.

D. impurities in the infused vancomycin preparation.

E. bacterial contamination of the vancomycin preparation.

A
270
Q

RACP 2003b Q66

QUESTION 66

A 37-year-old man with known human immunodeficiency virus (HIV) infection for 10 years presents with severe renal colic for which he has no prior history. Tests performed two weeks before have shown a normal full blood count and biochemistry screen (including serum calcium), CD4 positive (CD4+) T cell count of 36 cells/mm3, and HIV RNA concentration (viral load) of 83,000 copies/mL. His medications include zidovudine (AZT), lamivudine (3TC), indinavir, azithromycin and dapsone.

The most likely cause of his renal colic is:

A. HIV-associated opportunistic infection.

B. HIV-associated malignancy.

C. HIV-associated hypercalciuria.

D. adverse drug reaction.

E. unrelated to HIV infection or therapy.

A
271
Q

RACP 2003b Q68

QUESTION 68

Which one of the following confers the greatest risk for the development of fungal infection in a patient undergoing chemotherapy?

A. Prolonged granulocytopenia.

B. Use of steroids.

C. Long-term antibiotics.

D. Central venous lines.

E. Use of monoclonal antibodies.

A
272
Q

RACP 2003b Q75

A 45-year-old human immunodeficiency virus (HIV)-positive man presents with a two-month history of numbness in both feet. Examination reveals bilateral loss of light touch to mid-calf, reduced vibration sense at the ankle and absent ankle jerks. His CD4 positive (CD4+) T cell count is 250 cells/µL and HIV viral load is 5000 copies/mL. His current anti-retroviral (ARV) drug therapy is stavudine (D4T), lamivudine (3TC) and nevirapine.

The most likely diagnosis is:

A. Guillain-Barré syndrome.

B. HIV-induced neuropathy.

C. cytomegalovirus (CMV) neuropathy.

D. ARV-induced neuropathy.

E. vitamin B12 deficiency.

A
273
Q

RACP 2003b Q84

Ascaris lumbricoides is a common worm infestation in the developing world. Which one of the following syndromes would be most characteristic of this infection?

A. Eosinophilic meningitis.

B. Cholangitis.

C. Painless rectal bleeding.

D. Migratory rash.

E. Perianal itch.

A
274
Q

RACP 2002a Q9

QUESTION 9

Which one of the following is the most likely cause of vancomycin resistance in enterococci?

A. Changes in the enterococcal cell wall to prevent vancomycin binding.

B. Increased production of a beta-lactamase.

C. Alterations to enterococcal ribosomes to prevent vancomycin binding.

D. Increased breakdown of vancomycin.

E. Diminished affinity of penicillin-binding proteins (PBPs).

A
275
Q

RACP 2002a Q 22
Which one of the following is most likely to result in transmission of hepatitis C virus (HCV)?

A. An HCV-positive woman undergoing a normal vaginal delivery.

B. A blood transfusion.

C. Unprotected intercourse with an HCV RNA-positive partner.

D. Sharing household utensils with an HCV RNA-positive partner.

E. Illicit intravenous drug use.

A
276
Q

RACP 2002a Q40

Apart from antibiotics, which one of the following is the most beneficial treatment for acute oliguria (<0.5 mL/kg urine output in 24 hours) from ongoing hypotension associated with sepsis?

A. Intermittent frusemide.

B. Low (renal)-dose dopamine.

C. Frusemide infusion.

D. Prostacyclin infusion.

E. Adrenaline infusion.

A
277
Q

RACP 2002b Q9

QUESTION 9

A 37-year-old man with known human immunodeficiency virus (HIV) infection for 10 years presents with severe renal colic for which he has no prior history. Tests performed two weeks before have shown a normal full blood count and biochemistry screen (including serum calcium), CD4+ T lymphocyte count of 36 cells/mm3, HIV RNA concentration (viral load) of 83,000 copies/mL. His medications include zidovudine (AZT), lamivudine (3TC), indinavir, azithromycin and dapsone.

The most likely cause of his renal colic is:

A. HIV-associated opportunistic infection.

B. HIV-associated malignancy.

C. HIV-associated hypercalciuria.

D. adverse drug reaction.

E. unrelated to HIV infection or therapy.

A
278
Q

RACP 2002b Q10
Which one of the following is most likely to influence the outcome of chronic hepatitis C infection following treatment with combination ribavirin/interferon?

A. Age.

B. Viral genotype.

C. Viral load.

D. Mode of acquisition.

E. Gender.

A
279
Q

RACP 2002b Q22

QUESTION 22

A 30-year-old hospital staff member presents 24 hours after being exposed to a patient with confirmed measles. There is no prior history of measles or of vaccination against measles.

The most appropriate management is:

A. normal immunoglobulin.

B. ribavirin.

C. measles live attenuated vaccine.

D. aciclovir.

E. observation.

A
280
Q

RACP 2002b Q38

A 35-year-old man with human immunodeficiency virus (HIV) has been hospitalised for treatment of pneumocystis carinii pneumonia. He is alert and lucid until two days later when he becomes increasingly agitated, restless and irritable. He has become uncooperative with staff. He appears to be having auditory hallucinations and refers to his belief that his HIV infection has suddenly been cured. He is not sleeping, and has been disinhibited at night. He is forgetful and disoriented.

The most likely diagnosis is:

A. delirium.

B. HIV dementia.

C. mania.

D. acute schizophrenia.

E. major depression.

A
281
Q

RACP 2002b Q60

A 54-year-old man with methicillin-resistant Staphylococcus aureus (MRSA) sternal osteomyelitis is receiving ongoing treatment with vancomycin. Shortly after administration of his vancomycin dose, he develops diffuse erythema and hypotension.

The most likely cause is:

A. hypersensitivity to vancomycin.

B. rapid intravenous infusion of vancomycin.

C. endotoxin release.

D. impurities in the infused vancomycin preparation.

E. bacterial contamination of the vancomycin preparation.

A
282
Q

RACP 2002b Q76

QUESTION 76

A 56-year-old previously well man presents to the emergency department with a three-day history of headaches, fever, chills and rigors. On examination he has focal neurological signs. A computed tomography (CT) scan is performed and is shown below.

The most appropriate empirical antimicrobial therapy is:

A. ceftriaxone, metronidazole and aciclovir.

B. ceftriaxone, metronidazole and benzylpenicillin.

C. ceftriaxone, metronidazole and rifampicin.

D. ceftriaxone, metronidazole and vancomycin

A
283
Q

RACP 2002b Q87

Which one of the following modes of transmission of hepatitis B virus (HBV) is most likely to result in chronic carrier status in the non-immunised recipient?

A. A bite to a two-year-old by a chronic carrier.

B. Blood transfusion from a chronic carrier to a 60-year-old.

C. Horizontal transmission of HBV from a chronic carrier to his/her five-year-old sibling.

D. Needle-stick injury to a 21-year-old from a contaminated needle.

E. Vertical transmission of HBV from a pregnant carrier to her newborn baby.

A
284
Q

RACP 2002b Q99
A patient presents with the rash shown in the photograph below and is found to have a rapid plasma reagin (RPR) test titre of 1:256 and a positive fluorescent treponemal antibody absorbed (FTA-ABS) test.
The most appropriate treatment is:

A. doxycycline 100 mg twice daily for 14 days (oral).

B. benzylpenicillin 1.8 g sixth hourly for 10 days (intravenous).

C. benzathine penicillin 1.8 g as a single dose (intramuscular).

D. procaine penicillin 1g daily for 10 days (intramuscular).

E. ceftriaxone 250 mg daily for 5 days (intramuscular).

A
285
Q

RACP 2001a Q11

A number of patients in a ward have been found to be colonised with vancomycin-resistant enterococci. What would be the best laboratory test to determine if transmission from one patient to another has occurred?

A. Comparison of antibiotic susceptibility (antibiogram).

B. Polymerase chain reaction (PCR) for the van A and van B genes.

C. Enterococcal serology.

D. Pulse-field gel electrophoresis.

E. Speciation (e.g. faecalis vs. faecium).

A
286
Q

RACP 2001a Q23

Question 23

An 18-year-old woman presents with a two-week history of cervical lymphadenopathy, fever, fatigue and myalgia. A Paul-Bunnell test (heterophile antibody) has been negative on two occasions, a week apart. Full blood examination reveals a lymphocytosis with many atypical lymphocytes.

The most likely diagnosis is acute:

A. Epstein-Barr virus (EBV) infection.

B. streptococcal pharyngitis.

C. toxoplasmosis.

D. cytomegalovirus (CMV) infection.

E. human immunodeficiency virus (HIV) infection.

A
287
Q

RACP 2001b Q42

Which one of the following antibiotics is least active against pathogenic anaerobic bacteria (e.g. Bacteroides fragilis, Clostridium perfringens)?

A. Ticarcillin plus clavulanate.

B. Metronidazole.

C. Clindamycin.

D. Ceftriaxone.

E. Chloramphenico

A
288
Q

RACP 2001b Q54

The mechanism of penicillin resistance in pneumococcal infections is most likely to be due to which one of the following?

A. Plasmid-mediated beta lactamase production.

B. Chromosome-mediated beta lactamase production.

C. Plasmid-mediated penicillinase production.

D. Decreased affinity of penicillin-binding proteins.

E. Altered permeability to penicillin.

A
289
Q

RACP 2001a Q58

Genetically determined variation in which one of the following cell surface molecules is capable of conferring the greatest resistance to human immunodeficiency virus (HIV) infection?

A. Mannose-binding lectin.

B. Chemokine receptor CCR5.

C. Chemokine receptor CXCR4.

D. Human leucocyte antigen (HLA) class I.

E. Human leucocyte antigen (HLA) class II.

A
290
Q

RACP 2001a Q66

Which one of the following pharmacokinetic parameters is most important in the optimal prescribing of beta-lactam antibiotics?

A. The peak concentration/mean inhibitory concentration (MIC) ratio.

B. Area under the plasma concentration versus time curve (AUC).

C. The AUC/MIC ratio.

D. Time above the MIC.

E. Both AUC/MIC ratio and peak concentration/MIC ratio.

A
291
Q

RACP 2001b Q22

Question 22

A 54-year-old diabetic woman with chronic renal impairment presents with peritonitis secondary to a perforated diverticulum. Blood cultures grew Enterobacter species. The organism tests sensitive to ceftriaxone. After laparotomy, lavage and commencement of intravenous ceftriaxone and metronidazole, the patient makes no improvement. A computed tomography (CT) scan of the abdomen is performed and reveals no collection. Blood culture repeated three days later again grows the same Enterobacter species.

Which one of the following is the most appropriate next step?

A. Continuation of current therapy.

B. Laparotomy.

C. Cessation of ceftriaxone, continuation of metronidazole and addition of ampicillin and gentamicin.

D. Cessation of current antibiotics and commencement of meropenem.

E. Addition of vancomycin.

A
292
Q

RACP 2001b Q25

A 57-year-old HIV (human immunodeficiency virus)-positive man presents with a six-month history of gradual enlargement of the abnormality shown in the photographs above. Investigations reveal a CD4+ T cell count of 30/µL and a plasma HIV RNA concentration (viral load) of 158,000 copies/mL. Current medications are four antiretroviral drugs (zidovudine, lamivudine, ritonavir and saquinavir), antimicrobial prophylactic agents (cotrimoxazole, azithromycin) and amitriptyline. Routine blood count and biochemistry screen are normal except for an elevated fasting blood sugar level of 11.0 mmol/L [3.9-5.5].

Which one of the following represents the most likely pathogenesis for the abnormality shown in the photographs?

A. Opportunistic infection.

B. Malignancy.

C. Cortisol excess.

D. Drug toxicity.

E. Autoimmune inflammation.

A
293
Q

RACP 2001b Q34

Which one of the following modes of transmission of hepatitis B virus (HBV) is most likely to result in chronic carrier status in the non-immunised recipient?

A. A bite to a two-year-old by a chronic carrier.

B. Blood transfusion from a chronic carrier to a 60-year-old.

C. Horizontal transmission of HBV from a chronic carrier to his/her five-year-old sibling.

D. Needle-stick injury to a 21-year-old from a contaminated needle.

E. Vertical transmission of HBV from a pregnant carrier to her newborn baby.

A
294
Q

RACP 2001b Q41

A 32-year-old man with known human immunodeficiency virus (HIV) infection returns for routine assessment. Relevant history includes a documented seroconversion illness five years previously but no other features. He has not received antiretroviral therapy.

Physical examination reveals generalised lymphadenopathy but no other abnormalities. Investigations show normal full blood count and biochemistry screen. CD4+ T cell count is 560/µL with an undetectable HIV RNA concentration (viral load).

Which one of the following therapeutic options is the most appropriate?

A. Cotrimoxazole prophylaxis.

B. Azithromycin prophylaxis.

C. Zidovudine (AZT) and lamivudine (3TC).

D. AZT, 3TC and a protease inhibitor.

E. No therapy

A
295
Q

RACP 2001b Q49

Question 49

The optimal treatment for meningitis due to penicillin-resistant (minimum inhibitory concentration (MIC) >1mg/L) Streptococcus pneumoniae is:

A. high-dose ceftriaxone.

B. high-dose penicillin.

C. vancomycin and rifampicin.

D. cotrimoxazole and high-dose penicillin.

E. imipenem

A
296
Q

RACP 2001b Q63

Question 63

A 35-year-old man returns from a holiday in Africa. He has been taking mefloquine antimalarial prophylaxis until two weeks after his return to Australia. Eight weeks later he presents with fever and rigors. A blood film examination demonstrates Plasmodium vivax in 1% of red cells.

Which one of the following is the most appropriate management?

A. A three-day course of chloroquine followed by a seven-day course of doxycycline.

B. A three-day course of doxycycline.

C. A single dose of mefloquine followed by a 14-day course of primaquine.

D. A three-day course of chloroquine followed by a 14-day course of primaquine.

E. A three-day course of intravenous quinine followed by a seven-day course of oral doxycycline.

A
297
Q

RACP 2001b Q71

Question 71

The effect of neuraminidase inhibitors (such as zanamivir) when given within 24 hours of onset of symptoms on the course of influenza infection is best described as:

A. complete suppression of influenza symptoms within a day of commencing treatment.

B. reduction in the severity of influenza symptoms with no effect on duration of illness.

C. reduction in the severity of influenza symptoms and shortening of the duration of symptoms by one to two days.

D. reduction in the duration of illness by three to five days, with no effect on the severity of symptoms.

E. no effect on the course of the illness itself, but a marked reduction in the infectivity of the individual.

A
298
Q

RACP 2001b Q75

A 25-year-old man presents with dysuria and frequency after a trip to South-East Asia. A urethral swab is obtained and a Gram stain is performed (shown in the photograph below).

Which one of the following would be the optimal antibiotic treatment for this man?

A. Ceftriaxone 250 mg (intramuscularly, as a single dose).

B. Ciprofloxacin 500 mg and azithromycin 1 g (both orally, as a single dose).

C. Amoxicillin 3 g, probenecid 1 g and azithromycin 1 g (all orally, as a single dose).

D. Amoxicillin 3 g, probenecid 1 g (both orally, as a single dose) and doxycycline 100 mg (orally, 12 hourly for 10 days).

E. Benzathine penicillin 1.8 g (intramuscularly, as a single dose).

A
299
Q

RACP 2001b Q82

A 45-year-old man with known human immunodeficiency virus (HIV) infection presents with painful swallowing (odynophagia). He has not had any AIDS (acquired immune deficiency syndrome)-defining illnesses. He takes no medication.

An upper endoscopy is performed. A photograph of the mid-oesophagus is shown below. Biopsies are taken.

The most appropriate treatment at this stage is oral:

A. aciclovir.

B. fluconazole.

C. nystatin.

D. aciclovir and fluconazole.

E. aciclovir and nystatin.

A
300
Q

RACP 2001b Q97

A chest X-ray and a Ziehl-Nielsen (ZN) stain of sputum from a patient who is about to commence appropriate therapy are shown above. The patient is isolated with respiratory precautions. How long should the patient stay in respiratory isolation?

A. Until the patient is smear-negative (ZN stain-negative).

B. Two to three weeks.

C. Until the patient is culture-negative.

D. Until therapy is commenced.

E. Until asymptomatic.

A
301
Q

RACP 1999
Question 4

In which one of the following circumstances is hepatitis C transmission most likely to occur?

A. Intravenous drug use.

B. Unprotected sexual activity with multiple partners.

C. Vaginal delivery from a seropositive but polymerase chain reaction (PCR) negative mother.

D. Breastfeeding where the mother is seropositive and PCR positive.

E. Vaginal delivery from a seropositive, PCR positive mother.

A
302
Q

RACP 1999 Question 9

A Streptococcus pneumoniae isolate with high-level resistance to penicillin (MIC >2 µg/L) is most likely to be sensitive to which one of the following antibiotics?

A. Erythromycin.

B. Chloramphenicol.

C. Cefotaxime.

D. Vancomycin.

E. Ciprofloxacin.

A
303
Q

RACP 1999 Question 16

Which one of the following best describes hepatitis E virus infection?

A. Enteric transmission, acute self-limiting course, increased mortality in pregnant women.

B. Parenteral transmission, chronic course, mortality due to chronic liver disease.

C. Mode of transmission unknown, chronic course, mortality due to chronic liver disease.

D. Parenteral transmission, acute self-limiting course, increased mortality in pregnant women.

E. Mode of transmission unknown, chronic course, does not cause clinically significant illness.

A
304
Q

RACP 1999 Question 22

A 65-year-old woman is bitten by her cat on the dorsal aspect of the right middle finger. Twenty-four hours later, she develops chills and rigors and an abscess is aspirated. A bipolar staining, gram-negative coccobacillus grows from the culture.

The organism most likely to be responsible is:

A. Staphylococcus aureus.

B. Pseudomonas aeruginosa.

C. Bartonella henselae.

D. Bacteroides species.

E. Pasteurella multicida.

A
305
Q

RACP 1999 Question 25

The virus activated by immunosuppression in transplant patients that is least likely to be oncogenic is:

A. human herpes virus 8.

B. cytomegalovirus.

C. Epstein-Barr virus.

D. human papilloma virus.

E. hepatitis B virus.

A
306
Q

RACP 1999 Question 36

A 28-year-old man has a neutrophil count of 0.2 x 109/L [2.0-7.5] following induction chemotherapy for acute myeloid leukaemia. It has been three days since the chemotherapy was completed and he has been noted to be febrile with a temperature of 38.2°C. He is clinically well with stable vital signs. He has a Hickman catheter in situ which has become tender and erythematous along its subcutaneous tract. Chest X-ray is clear.

Which one of the following is the most likely cause of the fever?

A. Gram-positive bacteria.

B. Gram-negative bacteria.

C. Anaerobic bacteria.

D. Fungus.

E. Virus.

A
307
Q

RACP 1999 Question 42

A 23-year-old man with a four-year history of human immunodeficiency virus (HIV) infection, has a platelet count of 70 x 109/L [150-400]. He discontinued anti-retroviral treatment three months ago.

The most likely cause of his thrombocytopenia is:

A. enhanced clearance of platelets by splenic macrophages.

B. concurrent Pneumocystis carinii infection.

C. HIV infection of megakaryocytes.

D. previous treatment with reverse transcriptase inhibitors.

E. concurrent parvovirus infection.

A
308
Q

RACP 1999 Question 58

The C-C chemokine receptor 5 (CCR5) is the major co-receptor, with CD4, for macrophage tropic strains of the human immunodeficiency virus type 1 (HIV-1).

A 32 base pair deletion mutation in the CCR5 gene (CCR5-∆32) has been detected in 20% of Australians of northern European origin.

With respect to the CCR5-∆32 mutation, which one of the following is incorrect?

A. Homozygotes (∆32/∆32) occur with a prevalence of approximately 1% amongst Australians of

northern European origin.

B. Homozygotes (∆32/∆32) fail to express functional CCR5 on the surface of macrophages.

C. Homozygotes (∆32/∆32) are almost uninfectable with HIV-1.

D. Homozygotes (∆32/∆32) appear to have normal immune function due to redundancy in the

chemokine/chemokine receptor network.

E. There is no benefit associated with heterozygosity (∆32/wild type) with respect to the frequency or clinical course of HIV infection.

A
309
Q

RACP 1999 Question 66

Azithromycin is effective as a single oral dose in the treatment of Chlamydia trachomatis infection, in contrast to doxycycline which requires multiple doses over seven days to achieve similar cure rates.

Which property of azithromycin best explains this effect?

A. Lower mean bactericidal concentration.

B. Higher serum concentration.

C. Longer serum half-life.

D. Longer intracellular half-life.

E. Post-antibiotic effect

A
310
Q

RACP 1999 Question 70

Superantigens capable of inducing the toxic shock syndrome (such as the Staphylococcus aureus enterotoxins, SEA and SEB) stimulate proliferation in a sizeable proportion of T lymphocytes.

This effect is best explained by:

A. direct binding of the superantigen to the β chain of the T cell receptor (TcR).

B. direct binding of the superantigen to MHC class II.

C. cross-linking of MHC class II to the β chain of the TcR by the superantigen.

D. presentation of antigenic peptides of SEA and SEB by MHC class II to the TcR.

E. presentation of antigenic peptides of SEA and SEB by MHC class I to the TcR.

A
311
Q

RACP 1999b Q4
A 72-year-old patient on total parenteral nutrition (TPN) becomes unwell and develops a fever. A blood culture is reported as growing Candida krusei.

In addition to the removal of the central line, which one of the following management strategies is most appropriate?

A. Treatment with ketoconazole.

B. Treatment with amphotericin B.

C. Treatment with 5-Flucytosine.

D. Repeat blood cultures.

E. Observation only.

A
312
Q

RACP 1999b Q 8
Question 8

A recommended initial therapy for adults with a clinical diagnosis of bacterial meningitis prior to specific identification of a bacterial pathogen is ceftriaxone (or cefotaxime) plus benzyl penicillin.

Which one of the following represents the most important reason for the addition of penicillin to the antibiotic regimen?

A. Treatment for Listeria monocytogenes.

B. Treatment for Neisseria meningitidis.

C. To obtain synergy with the cephalosporin against Streptococcus pneumoniae isolates which are less sensitive to penicillin.

D. To include a “narrow spectrum” antibiotic in the regimen that can be continued as sole therapy after the identification of susceptible pathogens.

E. Treatment for Leptospira sp

A
313
Q

RACP 1999b Q13

Question 13

A 23-year-old healthy female presents with urinary frequency. Urine microscopy shows numerous leucocytes, erythrocytes and clumps of cocci. Culture reveals 104 cfu/ml of Staphylococcus saprophyticus.

The most appropriate next step would be to:

A. commence flucloxacillin (or dicloxacillin).

B. repeat urine culture.

C. commence amoxycillin.

D. test urine for nitrites.

E. commence ampicillin and gentamicin.

A
314
Q

RACP 1999b Q16

A 60-year-old man presents with a Streptococcus bovis endocarditis which is adequately diagnosed and treated.

Which one of the following is the next most appropriate investigation?

A. Colonoscopy.

B. Gallium scan.

C. Iron studies.

D. Small bowel series.

E. Human immunodeficiency virus (HIV) antibody test.

A

Answer A

315
Q

RACP 1999b Q21

A 34-year-old man with a history of multiple subcutaneous abscesses has had a painful, red, swollen area in the right thigh for five days and low-grade fever. Aspiration and culture of pus from a fluctuant abscess of the thigh reveals Bacteroides fragilis, Staphylococcus aureus, Escherichia coli, and Enterobacter aerogenes.

Which one of the following underlying conditions is most likely?

A. Hypogammaglobulinaemia.

B. Polymorphonuclear cell defect.

C. Acquired immunodeficiency syndrome (AIDS).

D. Self inoculation.

E. Diabetes.

A
316
Q

RACP 1999b Q28

Question 28

A 31-year-old man who has been HIV (human immunodeficiency virus)-positive for some years and who is on zidovudine, presents with acute left-sided chest pain and dyspnoea. He has smoked since the age of 17, but denies having asthma.

Physical examination shows a tall, thin man in mild respiratory distress with evidence of weight loss. His pulse rate is 120/minute and his respiratory rate is 28/minute. There are decreased breath sounds on the left side. His temperature is 38.6°C. Oxygen saturation on room air is 90% and his chest X-ray reveals a 45% pneumothorax on the left side.

The most likely cause for the spontaneous pneumothorax in this patient is:

A. smoking related disease.

B. HIV infection.

C. Streptococcus pneumoniae infection.

D. Pneumocystis carinii infection.

E. anti-retroviral therapy.

A
317
Q

RACP 1999b Q35

A 24-year-old woman consults you after a needle stick injury. She pricked her finger while collecting blood. The wound bled initially and she washed it. The source patient is HCV (hepatitis C antibody)-positive, HIV (human immunodeficiency virus)-negative and HBsAg (hepatitis B surface antigen)-negative.

In addition to acute and convalescent serology and regular liver function tests, which one of the following is the best management strategy?

A. Prophylaxis with pooled gamma globulin.

B. Treatment with interferon α if hepatitis occurs.

C. Treatment with lamivudine if hepatitis occurs.

D. Prophylaxis with lamivudine.

E. Prophylaxis with interferon α.

A
318
Q

RACP 1999b Q61

A 45-year-old man presents with acute respiratory distress and is noted to have oral candidiasis. Chest X-ray shows bilateral alveolar infiltrates. Arterial PaO2 on air is 55 mmHg.

A silver-stained preparation of his sputum is shown below.

Which one of the following is the most appropriate initial therapy?

A. Intravenous itraconazole.

B. Intravenous itraconazole and oral prednis(ol)one.

C. Intravenous amphotericin.

D. Intravenous cotrimoxazole.

E. Intravenous cotrimoxazole and oral prednis(ol)one.

A
319
Q

RACP 1999b Q74

A 60-year-old woman presented to the Accident and Emergency Department feeling unwell with a temperature of 38.5°C. Her pulse rate was 85/minute and regular and her blood pressure was 160/90 mmHg. Clinical examination revealed splenomegaly 2 cm below the left costal margin and petechiae of the lower limbs. Full blood examination revealed:

Blood film showed numerous myeloblasts. Chest X-ray is unremarkable. Cultures of urine and blood are taken.

How would the fever be best managed?

A. Ceftazidime and amphotericin.

B. Vancomycin and amphotericin.

C. Vancomycin and gentamicin.

D. Ceftazidime and gentamicin.

E. Await culture results before commencing antibiotics.

A
320
Q

RACP 1999b Q77

A 23-year-old woman has returned from a two-month trip to Burma with an acute febrile illness and has become obtunded in the last four hours. Her peripheral blood smear is shown.

Which one of the following is the most appropriate initial treatment?

A. Quinine.

B. Quinidine.

C. Doxycycline.

D. Chloroquine.

E. Mefloquine

A

Answer - NONE from the options

IV artesunate for P falciparum causing cerebral malaria

321
Q

RACP 1999b Q90

In an asymptomatic person with human immunodeficiency virus (HIV) infection, which one of the following is the best predictor of future rate of decline of immune function?

A. Serum β2 microglobulin concentration.

B. CD4+ lymphocyte count.

C. CD8+ lymphocyte count.

D. p24 antigenaemia.

E. Plasma HIV RNA concentration.

A

Answer B

The CD4+ T cell count is the most important laboratory indicator of immune function in HIV-infected patients.

322
Q

RACP 1999b Q99

A 35-year-old man became unwell while returning to Perth from a holiday in Johannesburg. On arrival, he has clinical signs of meningitis. The appearance of a Gram stain of his cerebrospinal fluid is shown.

The most appropriate initial intravenous antibiotic therapy is:

A. penicillin.
B. ceftriaxone.
C. ceftazidime.
D. ceftriaxone and vancomycin.
E. clindamycin and rifampicin.

A

Answer B

Image shows Gram negative diplococci likely Neisseria meningitidis (meningococcal meningitis),
Treatment including in patients with immediate nonsevere or delayed nonsevere hypersensitivity to penicillins, use:

ceftriaxone 2 g (child 1 month or older: 50 mg/kg up to 2 g) intravenously, 12-hourly for 5 days
OR
cefotaxime 2 g (child: 50 mg/kg up to 2 g) intravenously, 6-hourly for 5 days.

If susceptibility to benzylpenicillin is confirmed and the patient is not hypersensitive to penicillin, de-escalate therapy and use:

benzylpenicillin 2.4 g (child: 60 mg/kg up to 2.4 g) intravenously, 4-hourly for 5 days.

For penicillin severe hypersensitivity
ciprofloxacin 400 mg (child: 10 mg/kg up to 400 mg) intravenously, 8-hourly for 5 days

323
Q

170.Pulmonary fibrosis.
A. Azithromycin.
B. Cefaclor.
C. Cefepime.
D. Ciprofloxacin.
E. Clindamycin.
F. Flucloxacillin.
G. Nitrofurantoin.
H. Rifampicin.

A

Answer G Nitrofurantoin

324
Q

RACP 2013b Q73

Question 73
Patients who undergo allogeneic stem cell transplants are susceptible to invasive mould infections including Aspergillus fumigatus, Rhizopus and Scedosporium. Which host factor is most responsible for susceptibility to these invasive infections?

A. Corticosteroids
B. Previous pulmonary infections
C. Graft versus host disease
D. Diabetes mellitus
E. Prolonged neutropaenia

A

Answer C

Invasive fungal infections are common in selected hematopoietic cell transplant (HCT) recipients, such as allogeneic HCT recipients during the pre-engraftment period and in those with severe graft-versus-host disease and cause substantial morbidity and mortality.

325
Q

RACP 2013b Q78

Question 78
A 25yo man has recently been to Thailand (returned 1 week ago). He has never been vaccinated in his life. For the last 3 days, he has experienced a maculopapular rash (pictured), conjunctivitis and fever. What is the diagnosis?

A. Dengue fever
B. Influenza
C. Measles
D. Tuberculosis
E. Typhoid

A

Answer C Measles

326
Q

RACP 2012b Q94

QUESTION 94
In patients treated with trimethoprim/sulfamethoxazole (co-trimoxazole), what is the most common biochemical abnormality?

A. Hyperkalaemia
B. Hypernatraemia
C. Hypophosphataemia
D. Hypomagnasaemia
E. Hyperuricaemia

A

Answer A

Electrolyte disorders, particularly hyperkalemia and hyponatremia can be detected in patients given TMP-SMX. These disorders are more frequent in patients given high doses, but can also be detected after low-dose administration.